{"id": 274, "year": 2016, "question_id_specific": 73, "full_question": "Un hombre de 52 a\u00f1os sin enfermedades concomitantes, acude a urgencias por melenas de 24 horas de evoluci\u00f3n sin repercusi\u00f3n hemodin\u00e1mica. Niega consumo de antiinflamatorios no esteroideos. El hematocrito es de 33% y el resto de la anal\u00edtica es normal. La endoscopia digestiva alta realizada de forma urgente a las 6 horas del ingreso muestra un est\u00f3mago normal, sin sangre ni restos hem\u00e1ticos y una \u00falcera excavada de 8 mm de di\u00e1metro en cara anterior del bulbo duodenal con \u00abvaso visible\u00bb en su base y sin sangrado activo. \u00bfCu\u00e1l de las siguientes afirmaciones es cierta?", "full_answer": "Se trata de una \u00falcera de grado IIa (clasificaci\u00f3n de Forrest), con alto riesgo de recidiva. Por eso, se indica claramente el tratamiento endosc\u00f3pico y la hospitalizaci\u00f3n con tratamiento endovenoso con IBPs (en nuestro medio es habitualmente el omeprazol) durante al menos 72 horas. La cirug\u00eda est\u00e1 indicada en hemorragias masivas o refractarias al tratamiento m\u00e9dico.", "type": "APARATO DIGESTIVO", "options": {"1": "En la endoscopia inicial est\u00e1 indicado aplicar una terap\u00e9utica endosc\u00f3pica y posteriormente instaurar tratamiento endovenoso con dosis altas de un inhibidor de la bomba de protones. Esta estrategia ha demostrado reducir el riesgo de recidiva hemorr\u00e1gica y la mortalidad.", "2": "En la endoscopia inicial, dada la ausencia de sangrado activo, no est\u00e1 indicado aplicar una terap\u00e9utica endosc\u00f3pica. Posteriormente, para reducir el riesgo de recidiva hemorr\u00e1gica, se deber\u00e1 instaurar tratamiento endovenoso con dosis altas de un inhibidor de la bomba de protones.", "3": "n la endoscopia inicial est\u00e1 indicado aplicar una terap\u00e9utica endosc\u00f3pica. No se ha podido demostrar que instaurar posteriormente tratamiento con dosis altas de un inhibidor de la bomba de protones aporte beneficio adicional alguno.", "4": "Dado que se trata de una \u00falcera complicada (hemorragia) la mejor opci\u00f3n terap\u00e9utica, una vez resuelto el episodio hemorr\u00e1gico, es una vagotom\u00eda y piloroplastia.", "5": NaN}, "correct_option": 1, "explanations": {"1": {"exist": true, "char_ranges": [[0, 282]], "word_ranges": [[0, 44]], "text": "Se trata de una \u00falcera de grado IIa (clasificaci\u00f3n de Forrest), con alto riesgo de recidiva. Por eso, se indica claramente el tratamiento endosc\u00f3pico y la hospitalizaci\u00f3n con tratamiento endovenoso con IBPs (en nuestro medio es habitualmente el omeprazol) durante al menos 72 horas."}, "2": {"exist": false, "char_ranges": [], "word_ranges": [], "text": ""}, "3": {"exist": false, "char_ranges": [], "word_ranges": [], "text": ""}, "4": {"exist": true, "char_ranges": [[283, 368]], "word_ranges": [[44, 56]], "text": "La cirug\u00eda est\u00e1 indicada en hemorragias masivas o refractarias al tratamiento m\u00e9dico."}, "5": {"exist": false, "char_ranges": [], "word_ranges": [], "text": ""}}} {"id": 233, "year": 2014, "question_id_specific": 104, "full_question": "En una mujer de 29 a\u00f1os se hallan en el curso de una anal\u00edtica rutinaria los siguientes par\u00e1metros: Hb 11 .5 g/dL, VCM 70 fl, HCM 28 pg, Ferritina 10 ng/mL, leucocitos 5.200/mm3, plaquetas 335.000/mm3. La exploraci\u00f3n f\u00edsica es normal. La exploraci\u00f3n mas indicada en esta situaci\u00f3n es:", "full_answer": "La causa m\u00e1s frecuente en mujer joven de anemia ferrop\u00e9nica microc\u00edtica es por p\u00e9rdidas ginecol\u00f3gicas cr\u00f3nicas.", "type": "HEMATOLOG\u00cdA", "options": {"1": "Exploraci\u00f3n ginecol\u00f3gica.", "2": "Estudio de sangre oculta en heces.", "3": "Estudio radiol\u00f3gico del aparato digestivo.", "4": "Electroforesis de hemoglobinas.", "5": "Test de Cooms."}, "correct_option": 1, "explanations": {"1": {"exist": true, "char_ranges": [[0, 111]], "word_ranges": [[0, 16]], "text": "La causa m\u00e1s frecuente en mujer joven de anemia ferrop\u00e9nica microc\u00edtica es por p\u00e9rdidas ginecol\u00f3gicas cr\u00f3nicas."}, "2": {"exist": false, "char_ranges": [], "word_ranges": [], "text": ""}, "3": {"exist": false, "char_ranges": [], "word_ranges": [], "text": ""}, "4": {"exist": false, "char_ranges": [], "word_ranges": [], "text": ""}, "5": {"exist": false, "char_ranges": [], "word_ranges": [], "text": ""}}} {"id": 315, "year": 2016, "question_id_specific": 136, "full_question": "Un hombre de 41 a\u00f1os acude a Urgencias por un cuadro de tres d\u00edas de evoluci\u00f3n de tumefacci\u00f3n y dolor en la rodilla derecha, con impotencia funcional y febricula Dos semanas antes hab\u00eda tenido un cuadro de diarrea autoiimitada En la exploraci\u00f3n existe derrame articular, por lo que se procede a realizar una artrocentesis y se obtienen 50 cc de l\u00edquido de color turbio, con viscosidad disminuida y los siguientes par\u00e1metros anal\u00edticos: leucocitos 40.000/microL (85% de neutr\u00f3filos), glucosa 40 mg/dL, ausencia de cristales, tinci\u00f3n de Gram: no se observan microorganismos. \u00bfCu\u00e1l de las siguientes afirmaciones sobre este paciente es INCORRECTA?:", "full_answer": "Nunca un gram o un cultivo negativos descartan la opci\u00f3n de etiolog\u00eda infecciosa, a\u00fan menos en posible artritis s\u00e9pticas. Todas las dem\u00e1s son verdaderas.", "type": "REUMATOLOG\u00cdA", "options": {"1": "Se debe iniciar tratamiento con cloxacilina y ceftriaxona en espera del resultado del cultivo del l\u00edquido.", "2": "Es aconsejable realizar aitrocentesis diarias para aliviar los sintomas y evitar la destrucci\u00f3n articular.", "3": "Si el cultivo es negativo, es probable que se trate de una artritis reactiva.", "4": "La negatividad de la tinci\u00f3n de Gram descarta que se trate de una artritis s\u00e9ptica.", "5": NaN}, "correct_option": 4, "explanations": {"1": {"exist": false, "char_ranges": [], "word_ranges": [], "text": ""}, "2": {"exist": false, "char_ranges": [], "word_ranges": [], "text": ""}, "3": {"exist": false, "char_ranges": [], "word_ranges": [], "text": ""}, "4": {"exist": true, "char_ranges": [[0, 121]], "word_ranges": [[0, 19]], "text": "Nunca un gram o un cultivo negativos descartan la opci\u00f3n de etiolog\u00eda infecciosa, a\u00fan menos en posible artritis s\u00e9pticas."}, "5": {"exist": false, "char_ranges": [], "word_ranges": [], "text": ""}}} {"id": 235, "year": 2014, "question_id_specific": 107, "full_question": "Hombre de 71 a\u00f1os de edad que presenta anal\u00edtica con pancitopenia severa sin presencia de c\u00e9lulas inmaduras y con estudio medular sugestivo de anemia apl\u00e1sica grave. \u00bfCu\u00e1l ser\u00eda el abordaje terap\u00e9utico fundamental?", "full_answer": "Anemia apl\u00e1sica grave y no candidato a transplante alog\u00e9nico por su edad. Antisupresores parece lo m\u00e1s id\u00f3neo.", "type": "HEMATOLOG\u00cdA", "options": {"1": "Tratamiento con metilprednisolona a dosis de 1 g/Kg/d\u00eda durante 5 d\u00edas.", "2": "Estudio de hermanos y si alguno es HLA compatible, trasplante alog\u00e9nico de progenitores hematopoy\u00e9ticos.", "3": "Terapia inmunosupresora con ciclosporina e inmunoglobulina antitimoc\u00edtica.", "4": "Soporte hemoter\u00e1pico.", "5": "Quimioterapia y si respuesta trasplante aut\u00f3logo de progenitores hematopoy\u00e9ticos."}, "correct_option": 3, "explanations": {"1": {"exist": false, "char_ranges": [], "word_ranges": [], "text": ""}, "2": {"exist": false, "char_ranges": [], "word_ranges": [], "text": ""}, "3": {"exist": true, "char_ranges": [[0, 110]], "word_ranges": [[0, 17]], "text": "Anemia apl\u00e1sica grave y no candidato a transplante alog\u00e9nico por su edad. Antisupresores parece lo m\u00e1s id\u00f3neo."}, "4": {"exist": false, "char_ranges": [], "word_ranges": [], "text": ""}, "5": {"exist": false, "char_ranges": [], "word_ranges": [], "text": ""}}} {"id": 429, "year": 2018, "question_id_specific": 114, "full_question": "Hombre de 34 a\u00f1o que consulta en el servicio de urgencias por disuria y quemaz\u00f3n en el meato uretral, con presencia de una secreci\u00f3n blanquecina y espesa que sale a trav\u00e9s del meato desde hace tres d\u00edas. \u00bfCu\u00e1l de las siguientes respuestas es falsa?", "full_answer": "No se necesita una prueba de confirmaci\u00f3n de curaci\u00f3n microbiol\u00f3gica para las personas con un diagn\u00f3stico de gonorrea urogenital o rectal sin complicaciones que se tratan con cualquiera de los reg\u00edmenes recomendados o alternativos; sin embargo, cualquier persona con gonorrea far\u00edngea que se trate con un r\u00e9gimen alternativo debe regresar 14 d\u00edas despu\u00e9s del tratamiento para una prueba de curaci\u00f3n utilizando cultivo o un test de amplificaci\u00f3n de \u00e1cidos nucleicos (TAAN). Si el TAAN es positivo, se debe de tratar de realizar un cultivo confirmatorio antes del retratamiento. En todos los cultivos positivos deben de realizarse pruebas de susceptibilidad antimicrobiana. Los s\u00edntomas que persisten despu\u00e9s del tratamiento deben evaluarse mediante cultivo para N. gonorrhoeae (con o sin TAAN simult\u00e1nea), y cualquier gonococo aislado debe analizarse para identificar la sensibilidad antimicrobiana (modificado de Sexually Transmitted Diseases Treatment Guidelines, 2015).", "type": "ENFERMEDADES INFECCIOSAS Y MICROBIOLOG\u00cdA", "options": {"1": "Hay que obtener una muestra del exudado uretral para realizar el diagn\u00f3stico etiol\u00f3gico.", "2": "Se realizar\u00e1 tratamiento emp\u00edrico con ceftriaxona intramuscular y azitromicina v\u00eda oral en dosis \u00fanicas, en espera de los resultados microbiol\u00f3gicos.", "3": "Hay que realizar antibiograma en los casos que se aisle gonorrhoeae.", "4": "A las 2-3 semanas de la administraci\u00f3n del tratamiento, hay que obtener nuevas muestras uretrales para confirmar la curaci\u00f3n microbiol\u00f3gica.", "5": NaN}, "correct_option": 4, "explanations": {"1": {"exist": false, "char_ranges": [], "word_ranges": [], "text": ""}, "2": {"exist": false, "char_ranges": [], "word_ranges": [], "text": ""}, "3": {"exist": false, "char_ranges": [], "word_ranges": [], "text": ""}, "4": {"exist": true, "char_ranges": [[245, 472]], "word_ranges": [[36, 71]], "text": "cualquier persona con gonorrea far\u00edngea que se trate con un r\u00e9gimen alternativo debe regresar 14 d\u00edas despu\u00e9s del tratamiento para una prueba de curaci\u00f3n utilizando cultivo o un test de amplificaci\u00f3n de \u00e1cidos nucleicos (TAAN)."}, "5": {"exist": false, "char_ranges": [], "word_ranges": [], "text": ""}}} {"id": 540, "year": 2021, "question_id_specific": 4, "full_question": "Un paciente de 40 a\u00f1os comienza a sentir molestias en la nalga derecha tras un viaje prolongado en coche. Desde ese momento no tolera la sedestaci\u00f3n prolongada por reaparici\u00f3n del dolor. Ocasionalmente siente entumecimiento en el miembro inferior derecho que desaparece al levantarse. Dispone de estudio de resonancia magn\u00e9tica y radiograf\u00eda de cadera en los que no se informan anomal\u00edas. El examen de la sensibilidad y los reflejos osteotendinosos no muestra alteraciones. Las maniobras de rotaci\u00f3n externa y la abducci\u00f3n resistidas, con la cadera en 90 grados de flexi\u00f3n, reproducen el dolor. El diagn\u00f3stico m\u00e1s probable es:", "full_answer": "El s\u00edndrome del piramidal se produce debido a la compresi\u00f3n o pinzamiento del nervio ci\u00e1tico por hipertrofia o contractura del m\u00fasculo piriforme. Los s\u00edntomas caracter\u00edsticos del s\u00edndrome del piramidal incluyen un dolor gl\u00fateo irradiando la cara posterior del muslo que puede acompa\u00f1arse de parestesias. Como desencadenante frecuente el paciente puede referir un largo periodo de sedestaci\u00f3n sobre una superficie dura previo al dolor. El caso cl\u00ednico expone la maniobra de PACE en la que el paciente resiste al examinador la rotaci\u00f3n externa y la abducci\u00f3n de la cadera a 90\u00ba de flexi\u00f3n. Se considera positivo si reproduce los s\u00edntomas en contexto de s\u00edndrome del piramidal.", "type": "TRAUMATOLOG\u00cdA", "options": {"1": "S\u00edndrome de choque f\u00e9moro-acetabular.", "2": "S\u00edndrome del tensor de la fascia lata.", "3": "Radiculopat\u00eda S1.", "4": "S\u00edndrome del piramidal.", "5": NaN}, "correct_option": 4, "explanations": {"1": {"exist": false, "char_ranges": [], "word_ranges": [], "text": ""}, "2": {"exist": false, "char_ranges": [], "word_ranges": [], "text": ""}, "3": {"exist": false, "char_ranges": [], "word_ranges": [], "text": ""}, "4": {"exist": true, "char_ranges": [[435, 674]], "word_ranges": [[63, 105]], "text": "El caso cl\u00ednico expone la maniobra de PACE en la que el paciente resiste al examinador la rotaci\u00f3n externa y la abducci\u00f3n de la cadera a 90\u00ba de flexi\u00f3n. Se considera positivo si reproduce los s\u00edntomas en contexto de s\u00edndrome del piramidal."}, "5": {"exist": false, "char_ranges": [], "word_ranges": [], "text": ""}}} {"id": 266, "year": 2014, "question_id_specific": 138, "full_question": "En un paciente con enfermedad pulmonar obstructiva cr\u00f3nica (EPOC) leve en fase estable se evidencia poliglobulia e insuficiencia respiratoria. En la radiograf\u00eda de t\u00f3rax no se aprecian alteraciones rese\u00f1ables. \u00bfCu\u00e1l de los siguientes procedimientos considerar\u00eda realizar en primer lugar para descartar la coexistencia de otras enfermedades que puedan justificar los hallazgos descritos?:", "full_answer": "El planteamiento es dudoso por falta de datos suficientes. Por frecuencia, la respuesta correcta podria ser la la Poligrafia respiratoria, sobre todo en el caso de que se trate de un paciente obeso, dato que desconocemos, pero podria ser correcta tambien la 2 (Ecocardiograma) en el caso de sospecha de cardiopatia asociada, menos frecuente pero tambien posible. La asociacion de EPOC con SAOS es una causa frecuente de deterioro de la EPOC, mayor del que le corresponderia por afectacion funcional.", "type": "NEUMOLOG\u00cdA", "options": {"1": "Poligraf\u00eda respiratoria.", "2": "Ecocardiograma.", "3": "Tomograf\u00eda computarizada tor\u00e1cica.", "4": "Gammagraf\u00eda pulmonar.", "5": "Punci\u00f3n de m\u00e9dula \u00f3sea."}, "correct_option": 1, "explanations": {"1": {"exist": true, "char_ranges": [[59, 137]], "word_ranges": [[9, 20]], "text": "Por frecuencia, la respuesta correcta podria ser la la Poligrafia respiratoria,"}, "2": {"exist": true, "char_ranges": [[227, 362]], "word_ranges": [[37, 57]], "text": "podria ser correcta tambien la 2 (Ecocardiograma) en el caso de sospecha de cardiopatia asociada, menos frecuente pero tambien posible."}, "3": {"exist": false, "char_ranges": [], "word_ranges": [], "text": ""}, "4": {"exist": false, "char_ranges": [], "word_ranges": [], "text": ""}, "5": {"exist": false, "char_ranges": [], "word_ranges": [], "text": ""}}} {"id": 13, "year": 2011, "question_id_specific": 150, "full_question": "Ni\u00f1o de de a\u00f1o y medio que acude a urgencias por presentar dolor abdominal e ictericia. En la exploraci\u00f3n se palpa una masa abdominal. Al realizar una ecograf\u00eda se observa la presencia de un quiste de col\u00e9doco \u00bfQu\u00e9 actitud terap\u00e9utica adoptaremos?", "full_answer": "Creo que la respuesta correcta es la 4 el quiste de col\u00e9doco es una patolog\u00eda rara pero su tratamiento es quir\u00fargico y consiste en la resecci\u00f3n del quiste y hepaticoyeyunosotm\u00eda. La colangioresonancia es necesaria para elegir al t\u00e9cnica adecuada para las cuatro clases de quiste de col\u00e9doco existentes (clasificaci\u00f3n de Todani).", "type": "CIRUG\u00cdA", "options": {"1": "Se realizara una punci\u00f3n percut\u00e1nea para lavado peritoneal para comprobar que el quiste contiene bilis para dejar un drenaje que mejore el dolor abdominal.", "2": "Se indicar\u00e1 una resecci\u00f3n radical de toda la v\u00eda bilair para sustiruirla por un asa de intestino.", "3": "Se de indicar\u00e1 una laparotom\u00eda exploradora y un drenaje del quiste y cuando remita la dilataci\u00f3n se retirar\u00e1 el drenaje.", "4": "Se realizar\u00e1 una colangioresonancia para delimitar el quiste y se indicar\u00e1 una laparotom\u00eda para la resecci\u00f3n del quiste y anastomsosis de la v\u00eda biliar.", "5": "Es necesaria la gammagraf\u00eda con HIDA para delimitar el quiste y poder realizar el drenaje percut\u00e1neo con seguridad."}, "correct_option": 4, "explanations": {"1": {"exist": false, "char_ranges": [], "word_ranges": [], "text": ""}, "2": {"exist": false, "char_ranges": [], "word_ranges": [], "text": ""}, "3": {"exist": false, "char_ranges": [], "word_ranges": [], "text": ""}, "4": {"exist": true, "char_ranges": [[0, 328]], "word_ranges": [[0, 51]], "text": "Creo que la respuesta correcta es la 4 el quiste de col\u00e9doco es una patolog\u00eda rara pero su tratamiento es quir\u00fargico y consiste en la resecci\u00f3n del quiste y hepaticoyeyunosotm\u00eda. La colangioresonancia es necesaria para elegir al t\u00e9cnica adecuada para las cuatro clases de quiste de col\u00e9doco existentes (clasificaci\u00f3n de Todani)."}, "5": {"exist": false, "char_ranges": [], "word_ranges": [], "text": ""}}} {"id": 299, "year": 2016, "question_id_specific": 104, "full_question": "Hombre de 46 a\u00f1os usuario de drogas por v\u00eda parenteral que acude a urgencias presentando fiebre con escalofr\u00edos, confusi\u00f3n mental, mialgias difusas y dolor intenso en mano izquierda de 24 horas de evoluci\u00f3n sin claros signos flog\u00f3ticos locales. No refiere ning\u00fan antecedente traum\u00e1tico. A la exploraci\u00f3n destaca: temperatura 38,9\u201cC, 120 latidos por minuto, frecuencia respiratoria 30/min, presi\u00f3n arterial 90/54 mm Hg. En la anal\u00edtica destaca una leucocitosis con desviaci\u00f3n izquierda (25.000 leucocitos/mm3, 80% neutr\u00f3filos); aumento de las cifras de creatinina (1,6 mg/dL) y de CK (138 U/L). De entre los siguientes, \u00bfcu\u00e1l es el diagn\u00f3stico m\u00e1s probable?:", "full_answer": "Tengo mis dudas entre la 1 y la 2; excepcionalmente C. botulinum puede infectar heridas traum\u00e1ticas o por punci\u00f3n en UDVP que utilizan hero\u00edna marr\u00f3n o causar sinusitis en cocain\u00f3manos. Por otra parte, el caso se puede referir a una sepsis grave con pobre manifestaci\u00f3n inicial a primera vista, con rabdomiolisis incipiente y dolor muscular, en este caso puede tratarse de una fascitis necrotizante, donde el germen m\u00e1s frecuentemente implicado es el S. pyogenes.", "type": "ENFERMEDADES INFECCIOSAS", "options": {"1": "Fascitis necrotizante estreptoc\u00f3cica.", "2": "Gangrena por Clostridium spp.", "3": "Celulitis por micobacterias.", "4": "Erisipela.", "5": NaN}, "correct_option": 1, "explanations": {"1": {"exist": true, "char_ranges": [[202, 398]], "word_ranges": [[33, 64]], "text": "el caso se puede referir a una sepsis grave con pobre manifestaci\u00f3n inicial a primera vista, con rabdomiolisis incipiente y dolor muscular, en este caso puede tratarse de una fascitis necrotizante,"}, "2": {"exist": true, "char_ranges": [[35, 185]], "word_ranges": [[9, 30]], "text": "excepcionalmente C. botulinum puede infectar heridas traum\u00e1ticas o por punci\u00f3n en UDVP que utilizan hero\u00edna marr\u00f3n o causar sinusitis en cocain\u00f3manos."}, "3": {"exist": false, "char_ranges": [], "word_ranges": [], "text": ""}, "4": {"exist": false, "char_ranges": [], "word_ranges": [], "text": ""}, "5": {"exist": false, "char_ranges": [], "word_ranges": [], "text": ""}}} {"id": 16, "year": 2011, "question_id_specific": 131, "full_question": "Hombre de 62 a\u00f1os con antecedentes de h\u00e1bito en\u00f3lico importante, portador de virus de hepatitis C, sigue tratamiento con Ibuprofeno por una tendinitis en hombro derecho, acude a su dermat\u00f3logo porque despues de pasar dos semanas de vacaciones en la playa observa la aparici\u00f3n de ampollas tensas en dorso de manos. A la exploraci\u00f3n ademas de localizaci\u00f3n y ligera hipertricosis malar. El diagn\u00f3stico m\u00e1s probable es:", "full_answer": "Porfiria Cut\u00e1nea Tarda: el 60% de los pacientes con PCT, son varones, muchos de ellos ingieren alcohol en exceso, las mujeres que la desarrollan suelen estar en tratamiento con f\u00e1rmacos que contienen Estr\u00f3genos. La mayoria son varones con indicios de sobrecarga de hierro, esta sobrecarga reduce la actividad del enzima uroporfirin\u00f3geno descarboxilasa, que lleva a la elevaci\u00f3n de uroporfirinas. Se han implicado en la precipitaci\u00f3n de la PCT adquirida las infecciones por VHC y VIH. Existe una forma hereditaria con patr\u00f3n AD. Los pacientes con PCT, presentan ampollas en piel fotoexpuesta, m\u00e1s frecuente en dorso de manos y cuero cabelludo. Adem\u00e1s de la fragilidad, pueden desarrollar hipertricosis, hiperpigmentaci\u00f3n, alopecia cicatricial e induraci\u00f3n esclerod\u00e9rmica.", "type": "DERMATOLOG\u00cdA", "options": {"1": "Epidermolisis ampollosa adquirida.", "2": "Porfiria cut\u00e1nea tarda.", "3": "Reacci\u00f3n fotot\u00f3xica.", "4": "Dermatitis de cont\u00e1cto.", "5": "Porfiria aguda intermitente."}, "correct_option": 2, "explanations": {"1": {"exist": false, "char_ranges": [], "word_ranges": [], "text": ""}, "2": {"exist": true, "char_ranges": [[0, 395]], "word_ranges": [[0, 59]], "text": "Porfiria Cut\u00e1nea Tarda: el 60% de los pacientes con PCT, son varones, muchos de ellos ingieren alcohol en exceso, las mujeres que la desarrollan suelen estar en tratamiento con f\u00e1rmacos que contienen Estr\u00f3genos. La mayoria son varones con indicios de sobrecarga de hierro, esta sobrecarga reduce la actividad del enzima uroporfirin\u00f3geno descarboxilasa, que lleva a la elevaci\u00f3n de uroporfirinas."}, "3": {"exist": false, "char_ranges": [], "word_ranges": [], "text": ""}, "4": {"exist": false, "char_ranges": [], "word_ranges": [], "text": ""}, "5": {"exist": false, "char_ranges": [], "word_ranges": [], "text": ""}}} {"id": 121, "year": 2012, "question_id_specific": 114, "full_question": "Mujer de 47 a\u00f1os de edad. Natural de Cochabamba (Bolivia), reside en Espa\u00f1a desde hace 8 a\u00f1os. A mediados de 2009 comienza con palpitaciones, disnea y edemas progresivos en extremidades inferiores. Es diagnosticada de insuficiencia cardiaca secundaria a miocardiopat\u00eda por enfermedad de Chagas. \u00bfCual es el agente etiol\u00f3gico de esta enfermedad end\u00e9mica en varios paises de Latinoam\u00e9rica?", "full_answer": "La respuesta correcta es la quinta. La causa de la enfermedad de Chagas es Trypanosoma cruzi.", "type": "MICROBIOLOG\u00cdA", "options": {"1": "Toxoplasma gondii.", "2": "Trypanosoma brucei.", "3": "Lehismania donovani.", "4": "Giardia lamblia.", "5": "Trypanosoma cruzi."}, "correct_option": 5, "explanations": {"1": {"exist": false, "char_ranges": [], "word_ranges": [], "text": ""}, "2": {"exist": false, "char_ranges": [], "word_ranges": [], "text": ""}, "3": {"exist": false, "char_ranges": [], "word_ranges": [], "text": ""}, "4": {"exist": false, "char_ranges": [], "word_ranges": [], "text": ""}, "5": {"exist": true, "char_ranges": [[36, 93]], "word_ranges": [[6, 16]], "text": "La causa de la enfermedad de Chagas es Trypanosoma cruzi."}}} {"id": 241, "year": 2014, "question_id_specific": 229, "full_question": "Mujer de 29 a\u00f1os, obesa, sin antecedentes de inter\u00e9s, que consulta por cefalea puls\u00e1til, bilateral, intensa, de un mes de evoluci\u00f3n, acompa\u00f1ada de diplopia horizontal y episodios de amaurosis monocular de segundos de duraci\u00f3n. La exploraci\u00f3n es normal, salvo por la presencia de papiledema bilateral. \u00bfCu\u00e1l de estas pruebas cree que le va a permitir confirmar plenamente su diagn\u00f3stico?", "full_answer": "Nuevamente nos presentan un caso para realizar un diagn\u00f3stico y luego, en este caso, realizar una prueba confirmatoria. Ahora el s\u00edntoma gu\u00eda es el papiledema bilateral que es indicativa de hipertensi\u00f3n intracraneal. Aunque no comenta se haya descartado una lesi\u00f3n ocupante de espacio, dado que la exploraci\u00f3n neurol\u00f3gica normal y con el resto de s\u00edntomas se deduce que se trata de una hipertensi\u00f3n intracraneal idiop\u00e1tica o pseudotumor cerebri. Entre los otros datos sugestivos de esta entidad est\u00e1n: mujer en edad reproductiva, obesidad, y diplopia (por afectaci\u00f3n de VI par craneal). Es caracter\u00edstica que la cefalea y la borrosidad visual se agraven s\u00fabitamente por las ma\u00f1anas al levantarse o al cambiar de postura produciendocegueras transitorias (Neurolog\u00eda, Zarranz). La punci\u00f3n lumbar presenta, de forma invariable, un incremento de presi\u00f3n de l\u00edquido cefalorraqu\u00eddeo, por lo cual la respuesta correcta es la 4.", "type": "NEUROLOG\u00cdA", "options": {"1": "Ecograf\u00eda de troncos supraa\u00f3rticos.", "2": "Resonancia magn\u00e9tica de cr\u00e1neo.", "3": "Electroencefalograma.", "4": "Punci\u00f3n lumbar.", "5": "Potenciales evocados visuales."}, "correct_option": 4, "explanations": {"1": {"exist": false, "char_ranges": [], "word_ranges": [], "text": ""}, "2": {"exist": false, "char_ranges": [], "word_ranges": [], "text": ""}, "3": {"exist": false, "char_ranges": [], "word_ranges": [], "text": ""}, "4": {"exist": true, "char_ranges": [[587, 920]], "word_ranges": [[90, 139]], "text": "Es caracter\u00edstica que la cefalea y la borrosidad visual se agraven s\u00fabitamente por las ma\u00f1anas al levantarse o al cambiar de postura produciendocegueras transitorias (Neurolog\u00eda, Zarranz). La punci\u00f3n lumbar presenta, de forma invariable, un incremento de presi\u00f3n de l\u00edquido cefalorraqu\u00eddeo, por lo cual la respuesta correcta es la 4."}, "5": {"exist": false, "char_ranges": [], "word_ranges": [], "text": ""}}} {"id": 576, "year": 2022, "question_id_specific": 103, "full_question": "Var\u00f3n de 30 a\u00f1os que consulta por cefaleas perioculares unilaterales que progresan r\u00e1pidamente hasta hacerse muy intensas. Las crisis de dolor duran entre 30-60 minutos, ocurren 2-3 veces al d\u00eda y se acompa\u00f1an de lagrimeo en el ojo que le duele y rinorrea unilateral. \u00bfCu\u00e1l es el tratamiento preventivo m\u00e1s adecuado?:", "full_answer": "Describen un caso caracter\u00edstico de cefalea en racimos. Es m\u00e1s frecuente en varones, el dolor es caracter\u00edsticamente periocular unilateral de gran intensidad, de duraci\u00f3n variable, generalmente minutos y asocia signos trigeminoauton\u00f3micos como la rinorrea e inyecci\u00f3n conjuntival unilateral. El tratamiento preventivo por excelencia es el verapamilo asociado a pauta descendente de corticoides, tal y como se expone en la respuesta 2.", "type": "NEUROLOG\u00cdA", "options": {"1": "Prednisona oral con retirada en 2-3 meses.", "2": "Prednisona oral con retirada en un mes, asociada a verapamilo oral a dosis de 80-180 mg cada 8 horas.", "3": "Carbamazepina a dosis de 600-1200 mg al d\u00eda.", "4": "Indometacina a dosis de 50 mg cada 8 horas durante un mes.", "5": NaN}, "correct_option": 2, "explanations": {"1": {"exist": false, "char_ranges": [], "word_ranges": [], "text": ""}, "2": {"exist": true, "char_ranges": [[0, 434]], "word_ranges": [[0, 61]], "text": "Describen un caso caracter\u00edstico de cefalea en racimos. Es m\u00e1s frecuente en varones, el dolor es caracter\u00edsticamente periocular unilateral de gran intensidad, de duraci\u00f3n variable, generalmente minutos y asocia signos trigeminoauton\u00f3micos como la rinorrea e inyecci\u00f3n conjuntival unilateral. El tratamiento preventivo por excelencia es el verapamilo asociado a pauta descendente de corticoides, tal y como se expone en la respuesta 2."}, "3": {"exist": false, "char_ranges": [], "word_ranges": [], "text": ""}, "4": {"exist": false, "char_ranges": [], "word_ranges": [], "text": ""}, "5": {"exist": false, "char_ranges": [], "word_ranges": [], "text": ""}}} {"id": 504, "year": 2020, "question_id_specific": 87, "full_question": "Ante un trastorno obsesivo-compulsivo (TOC) de inicio s\u00fabito en un ni\u00f1o de 9 a\u00f1os que adem\u00e1s presenta tics y corea como manifestaciones neurol\u00f3gicas se deber\u00eda descartar una infecci\u00f3n por:", "full_answer": "Nos presentan un caso de S\u00edndrome PANDAS, complicaci\u00f3n rara de una infecci\u00f3n producida por el estreptococo beta-hemol\u00edtico del grupo A.", "type": "PEDIATR\u00cdA", "options": {"1": "Estreptococo viridans.", "2": "Estreptococo alfa-hemol\u00edtico.", "3": "Estreptococo beta-hemol\u00edtico, grupo A.", "4": "Enterococo.", "5": NaN}, "correct_option": 3, "explanations": {"1": {"exist": false, "char_ranges": [], "word_ranges": [], "text": ""}, "2": {"exist": false, "char_ranges": [], "word_ranges": [], "text": ""}, "3": {"exist": true, "char_ranges": [[0, 135]], "word_ranges": [[0, 20]], "text": "Nos presentan un caso de S\u00edndrome PANDAS, complicaci\u00f3n rara de una infecci\u00f3n producida por el estreptococo beta-hemol\u00edtico del grupo A."}, "4": {"exist": false, "char_ranges": [], "word_ranges": [], "text": ""}, "5": {"exist": false, "char_ranges": [], "word_ranges": [], "text": ""}}} {"id": 367, "year": 2016, "question_id_specific": 119, "full_question": "Mujer de 38 a\u00f1os, no fumadora, diagnosticada previamente de rinitis at\u00f3pica que acude por cuadro de dos meses consistente en tos y sibilantes intermitentes de predominio nocturno. Rx de t\u00f3rax normal. Espirometr\u00eda dentro de la normalidad con prueba broncodilatadora negativa. \u00bfCu\u00e1l de las siguientes pruebas solicitar\u00eda a continuaci\u00f3n?", "full_answer": "Una espirometria normal y un test broncodilatador negativo puntualmente no excluye asma, y la cl\u00ednica de la paciente es altamente sugestiva de asma, por lo que un test de metacolina permitir\u00eda confirmar el diagnostico, al menos de hiperreactividad bronquial.", "type": "NEUMOLOG\u00cdA Y CIRUG\u00cdA TOR\u00c1CICA", "options": {"1": "Rx de senos paranasales.", "2": "TAC tor\u00e1cico.", "3": "Test de metacolina.", "4": "Prick-test.", "5": NaN}, "correct_option": 3, "explanations": {"1": {"exist": false, "char_ranges": [], "word_ranges": [], "text": ""}, "2": {"exist": false, "char_ranges": [], "word_ranges": [], "text": ""}, "3": {"exist": true, "char_ranges": [[0, 258]], "word_ranges": [[0, 39]], "text": "Una espirometria normal y un test broncodilatador negativo puntualmente no excluye asma, y la cl\u00ednica de la paciente es altamente sugestiva de asma, por lo que un test de metacolina permitir\u00eda confirmar el diagnostico, al menos de hiperreactividad bronquial."}, "4": {"exist": false, "char_ranges": [], "word_ranges": [], "text": ""}, "5": {"exist": false, "char_ranges": [], "word_ranges": [], "text": ""}}} {"id": 444, "year": 2018, "question_id_specific": 104, "full_question": "Mujer de 36 a\u00f1os que acude a la consulta por haber sufrido tres abortos espont\u00e1neos en el primer trimestre. No ha tenido ning\u00fan embarazo a t\u00e9rmino. Entre los estudios que solicitar\u00e1 en primer t\u00e9rmino, NO se incluye:", "full_answer": "La respuesta correcta es la 1. Esto es as\u00ed porque la histerosalpingograf\u00eda determina si hay alguna obstruccci\u00f3n de las trompas y, por ello, no ocurre la fecundaci\u00f3n. \u00c9ste no es el caso de nuestra paciente, donde s\u00ed ocurre la fecundaci\u00f3n ya que llega a implantarse incluso. El resto de pruebas diagn\u00f3sticas s\u00ed estar\u00edan incluidas.", "type": "GINECOLOG\u00cdA Y OBSTETRICIA", "options": {"1": "Histerosalpingografia.", "2": "Cariotipo en sangre perif\u00e9rica de ambos miembros de la pareja.", "3": "Determinaci\u00f3n de anticuerpos antifosfol\u00edpidos.", "4": "Ecografia vaginal.", "5": NaN}, "correct_option": 1, "explanations": {"1": {"exist": true, "char_ranges": [[0, 272]], "word_ranges": [[0, 46]], "text": "La respuesta correcta es la 1. Esto es as\u00ed porque la histerosalpingograf\u00eda determina si hay alguna obstruccci\u00f3n de las trompas y, por ello, no ocurre la fecundaci\u00f3n. \u00c9ste no es el caso de nuestra paciente, donde s\u00ed ocurre la fecundaci\u00f3n ya que llega a implantarse incluso."}, "2": {"exist": true, "char_ranges": [[273, 328]], "word_ranges": [[46, 54]], "text": "El resto de pruebas diagn\u00f3sticas s\u00ed estar\u00edan incluidas."}, "3": {"exist": true, "char_ranges": [[273, 328]], "word_ranges": [[46, 54]], "text": "El resto de pruebas diagn\u00f3sticas s\u00ed estar\u00edan incluidas."}, "4": {"exist": true, "char_ranges": [[273, 328]], "word_ranges": [[46, 54]], "text": "El resto de pruebas diagn\u00f3sticas s\u00ed estar\u00edan incluidas."}, "5": {"exist": false, "char_ranges": [], "word_ranges": [], "text": ""}}} {"id": 171, "year": 2013, "question_id_specific": 226, "full_question": "Paciente var\u00f3n de 26 a\u00f1os de edad, deportista de competici\u00f3n, es estudiado porque su hermano ha tenido una muerte s\u00fabita. Las pruebas complementarias detectan la existencia de una miocardiopat\u00eda hipertr\u00f3fica obstructiva. Se\u00f1ale la opci\u00f3n FALSA respecto a dicha patolog\u00eda:", "full_answer": "Tenemos un miocardio m\u00e1s grueso de lo debido, con predominio en el septo, y que adem\u00e1s est\u00e1 tan grueso que llega a obstruir la salida del VI, produciendo una estenosis \u201cfuncional\u201d con gradiente de presiones. Nuestro objetivo ser\u00e1, en primer lugar, \u201crelajar\u201d el miocardio: para ello nos conviene un betabloqueante, mientras que la digoxina, como buen inotropo positivo, aumenta el gradiente y est\u00e1 contraindicada. El deporte no es muy recomendable, dado el riesgo aumentado de muerte s\u00fabita que tiene esta patolog\u00eda. Y si tenemos un septo gordo, que protruye en la cavidad del VI, no nos conviene hacer una depleci\u00f3n de volumen, pues estar\u00edamos tambi\u00e9n aumentando el gradiente. En cuanto a la 5, es cierta: es conveniente realizar una exploraci\u00f3n cuidadosa y un ECG en los familiares de pacientes con hipertr\u00f3fica.", "type": "CARDIOLOG\u00cdA Y CIRUG\u00cdA CARDIOVASCULAR", "options": {"1": "El tratamiento con betabloqueantes ayuda a reducir los s\u00edntomas.", "2": "Se debe recomendar el abandono del deporte de competici\u00f3n.", "3": "La digoxina es \u00fatil para reducir el gradiente suba\u00f3rtico.", "4": "Los diur\u00e9ticos se deben evitar o utilizar con gran cuidado.", "5": "Dado el car\u00e1cter hereditario de la patolog\u00eda se debe realizar estudio de despistaje en el resto de familiares directos."}, "correct_option": 3, "explanations": {"1": {"exist": true, "char_ranges": [[208, 312]], "word_ranges": [[35, 50]], "text": "Nuestro objetivo ser\u00e1, en primer lugar, \u201crelajar\u201d el miocardio: para ello nos conviene un betabloqueante,"}, "2": {"exist": true, "char_ranges": [[413, 515]], "word_ranges": [[64, 81]], "text": "El deporte no es muy recomendable, dado el riesgo aumentado de muerte s\u00fabita que tiene esta patolog\u00eda."}, "3": {"exist": true, "char_ranges": [[314, 412]], "word_ranges": [[50, 64]], "text": "mientras que la digoxina, como buen inotropo positivo, aumenta el gradiente y est\u00e1 contraindicada."}, "4": {"exist": true, "char_ranges": [[516, 676]], "word_ranges": [[81, 108]], "text": "Y si tenemos un septo gordo, que protruye en la cavidad del VI, no nos conviene hacer una depleci\u00f3n de volumen, pues estar\u00edamos tambi\u00e9n aumentando el gradiente."}, "5": {"exist": true, "char_ranges": [[677, 813]], "word_ranges": [[108, 131]], "text": "En cuanto a la 5, es cierta: es conveniente realizar una exploraci\u00f3n cuidadosa y un ECG en los familiares de pacientes con hipertr\u00f3fica."}}} {"id": 555, "year": 2022, "question_id_specific": 51, "full_question": "Mujer de 55 a\u00f1os con diabetes tipo 1 diagnosticada a los 15 a\u00f1os que refiere n\u00e1useas y distensi\u00f3n abdominal, especialmente despu\u00e9s de las comidas. La evaluaci\u00f3n es compatible con una gastroparesia diab\u00e9tica. \u00bfCu\u00e1l de los siguientes medicamentos ser\u00eda el tratamiento m\u00e1s adecuado para mejorar sus s\u00edntomas?:", "full_answer": "Fue aprobada como tratamiento para la gastroparesia por la Food and Drug Administration (FDA) de los Estados Unidos en 1979 y sigue siendo el f\u00e1rmaco de primera l\u00ednea. Act\u00faa como procin\u00e9tico por su efecto antag\u00f3nico sobre la dopamina 2 de los receptores (D2) promoviendo el vaciamiento g\u00e1strico y, adem\u00e1s, se une al receptor 5-hidroxitriptamina 4 (serotonina 5-HT4) para estimular las v\u00edas nerviosas colin\u00e9rgicas en el est\u00f3mago. Fisiol\u00f3gicamente, acelera el tr\u00e1nsito intestinal al aumentar el tono y la amplitud de las contracciones g\u00e1stricas, incrementa la presi\u00f3n del esf\u00ednter esof\u00e1gico inferior y mejora la coordinaci\u00f3n antropiloroduodenal. Adem\u00e1s, este agente antiem\u00e9tico proporciona alivio a trav\u00e9s del antagonismo de receptores centrales y perif\u00e9ricos de la dopamina.", "type": "ENDOCRINOLOG\u00cdA", "options": {"1": "Lansoprazol.", "2": "Ondansetr\u00f3n.", "3": "Loperamida.", "4": "Metoclopramida.", "5": NaN}, "correct_option": 4, "explanations": {"1": {"exist": false, "char_ranges": [], "word_ranges": [], "text": ""}, "2": {"exist": false, "char_ranges": [], "word_ranges": [], "text": ""}, "3": {"exist": false, "char_ranges": [], "word_ranges": [], "text": ""}, "4": {"exist": true, "char_ranges": [[0, 167]], "word_ranges": [[0, 28]], "text": "Fue aprobada como tratamiento para la gastroparesia por la Food and Drug Administration (FDA) de los Estados Unidos en 1979 y sigue siendo el f\u00e1rmaco de primera l\u00ednea."}, "5": {"exist": false, "char_ranges": [], "word_ranges": [], "text": ""}}} {"id": 320, "year": 2016, "question_id_specific": 142, "full_question": "Hombre de 40 a\u00f1os, sin antecedentes de inter\u00e9s,que presenta artritis aguda de rodilla derecha de 5 d\u00edas de evoluci\u00f3n as\u00ed como febr\u00edcula de hasta 37,7\u00b0C. No refiere antecedente traum\u00e1tico. Presenta derrame articular, calor a la palpaci\u00f3n y limitaci\u00f3n funcional por el dolor. Se practica artrocentesis diagn\u00f3stica en la que se evidencian 30000 cels/m\u00edcroL con predominio de neutr\u00f3filos. En el examen con luz polarizada se descubren cristales romboidales y cil\u00edndricos con birrefringencia d\u00e9bilmente positiva. En la radiograf\u00eda simple de rodillas se descubren dep\u00f3sitos radiodensos en el menisco externo de la rodilla. Se\u00f1ale la respuesta FALSA:", "full_answer": "Es una condrocalcinosis. No existe tratamiento para esta enfermedad, \u00fanicamente tratamiento para los brotes (colchicina, AINEs o corticoides) aunque en ocasiones si los brotes son habituales se deja la colchicina mantenida. El alopurinol no ha demostrado ninguna eficacia en esta artropat\u00eda por microcristales, s\u00ed en la gota.", "type": "REUMATOLOG\u00cdA", "options": {"1": "El tratamiento inicial consistir\u00e1 en antiinflamatorios no esteroideos, as\u00ed como la aspiraci\u00f3n del l\u00edquido sinovial.", "2": "Es conveniente la realizaci\u00f3n de cultivo del l\u00edquido sinovial para descartar sinovitis infecciosa.", "3": "Una vez pasado el brote agudo habr\u00e1 que instaurar tratamiento con alopurinol para disminuir la incidencia de nuevos brotes de artritis en el futuro.", "4": "El diagn\u00f3stico m\u00e1s probable es una artritis aguda por dep\u00f3sito de cristales de pirofosfato c\u00e1lcico (pseudogota).", "5": NaN}, "correct_option": 3, "explanations": {"1": {"exist": true, "char_ranges": [[25, 141]], "word_ranges": [[3, 18]], "text": "No existe tratamiento para esta enfermedad, \u00fanicamente tratamiento para los brotes (colchicina, AINEs o corticoides)"}, "2": {"exist": false, "char_ranges": [], "word_ranges": [], "text": ""}, "3": {"exist": true, "char_ranges": [[224, 325]], "word_ranges": [[31, 47]], "text": "El alopurinol no ha demostrado ninguna eficacia en esta artropat\u00eda por microcristales, s\u00ed en la gota."}, "4": {"exist": true, "char_ranges": [[224, 325]], "word_ranges": [[31, 47]], "text": "El alopurinol no ha demostrado ninguna eficacia en esta artropat\u00eda por microcristales, s\u00ed en la gota."}, "5": {"exist": false, "char_ranges": [], "word_ranges": [], "text": ""}}} {"id": 617, "year": 2022, "question_id_specific": 119, "full_question": "Mujer de 13 a\u00f1os, sin antecedentes relevantes, con menarquia hace 3 meses, seguida desde los 10 a\u00f1os por escoliosis idiop\u00e1tica que ha empeorado. En la exploraci\u00f3n f\u00edsica presenta una giba de 7 grados en el test de Adams y en el escoliograma una curva toracolumbar T4-L1 de 35 grados de Cobb y un Risser 0. La actitud correcta a tomar ser\u00e1:", "full_answer": "Nos presentan el caso de una paciente de 13 a\u00f1os, con la menarquia hace 3 meses, una curva toracolumbar T4-L1 de 35\u00ba y un Risser 0. Como norma general podr\u00edamos decir que por debajo de los 50\u00ba el tratamiento suele ser conservador, existiendo dos opciones: observaci\u00f3n o uso de cors\u00e9 para enlentecer la evoluci\u00f3n. En este caso teniendo en cuenta el crecimiento remanente que queda (menarquia hace 3 meses y Risser 0) lo m\u00e1s indicado ser\u00eda utilizar cors\u00e9 (Respuesta 2 correcta).", "type": "TRAUMATOLOG\u00cdA", "options": {"1": "Recomendar nataci\u00f3n y revisi\u00f3n en tres meses.", "2": "Prescribir una ortesis tipo cors\u00e9.", "3": "Derivar a fisioterapia para elastificaci\u00f3n del raquis.", "4": "Revisar en 6 meses con una nueva radiograf\u00eda.", "5": NaN}, "correct_option": 2, "explanations": {"1": {"exist": false, "char_ranges": [], "word_ranges": [], "text": ""}, "2": {"exist": true, "char_ranges": [[0, 476]], "word_ranges": [[0, 81]], "text": "Nos presentan el caso de una paciente de 13 a\u00f1os, con la menarquia hace 3 meses, una curva toracolumbar T4-L1 de 35\u00ba y un Risser 0. Como norma general podr\u00edamos decir que por debajo de los 50\u00ba el tratamiento suele ser conservador, existiendo dos opciones: observaci\u00f3n o uso de cors\u00e9 para enlentecer la evoluci\u00f3n. En este caso teniendo en cuenta el crecimiento remanente que queda (menarquia hace 3 meses y Risser 0) lo m\u00e1s indicado ser\u00eda utilizar cors\u00e9 (Respuesta 2 correcta)."}, "3": {"exist": false, "char_ranges": [], "word_ranges": [], "text": ""}, "4": {"exist": false, "char_ranges": [], "word_ranges": [], "text": ""}, "5": {"exist": false, "char_ranges": [], "word_ranges": [], "text": ""}}} {"id": 146, "year": 2012, "question_id_specific": 231, "full_question": "Un lactante de 7 meses acude a urgencias derivado por su pediatra de zona por sospecha de enfermedad de Kawasaki. Refiere fiebre de hasta 39,5\u00baC de 5 d\u00edas de evoluci\u00f3n que no cede a pesar del tratamiento con amoxicilina pautado hace 72 horas por sospecha de faringoamigdalitis aguda. Durante las \u00faltimas 24 horas asocia exantema eritematoso ni pruriginoso de evoluci\u00f3n cefalocaudal y durante los \u00faltimos d\u00edas llama la atenci\u00f3n a sus padres la tos y una intensa conjuntivitis. No refieren rinorrea. A la exploraci\u00f3n cl\u00ednica presenta subcrepitantes en ambas bases pulmonares sin taquipnea ni signos de dificultad respiratoria. A la exploraci\u00f3n orofar\u00edngea se evidencian manchas blanquecinas en mucosa yugal. \u00bfCu\u00e1l es el diagn\u00f3stico m\u00e1s probable?", "full_answer": "La respuesta correcta es la 5. La pista est\u00e1 en las lesiones blanquecinas de la mucosa yugal que corresponden a las manchas de Koplik, que son patognom\u00f3nicas del sarampi\u00f3n, aunque como observaci\u00f3n dir\u00eda que suelen aparecer en la fase prodr\u00f3mica de la enfermedad y muchas veces ya no se ven cuando aparece el exantema. En este caso est\u00e1 describiendo un sarampi\u00f3n complicado con una neumon\u00eda. Muy oportuna esta pregunta en el momento actual.", "type": "PEDIATR\u00cdA", "options": {"1": "Enfermedad de Kawasaki.", "2": "Escarlatina.", "3": "Reacci\u00f3n al\u00e9rgica a la amoxicilina.", "4": "Mononucleosis infecciosa.", "5": "Sarampi\u00f3n."}, "correct_option": 5, "explanations": {"1": {"exist": false, "char_ranges": [], "word_ranges": [], "text": ""}, "2": {"exist": false, "char_ranges": [], "word_ranges": [], "text": ""}, "3": {"exist": false, "char_ranges": [], "word_ranges": [], "text": ""}, "4": {"exist": false, "char_ranges": [], "word_ranges": [], "text": ""}, "5": {"exist": true, "char_ranges": [[31, 171]], "word_ranges": [[6, 29]], "text": "La pista est\u00e1 en las lesiones blanquecinas de la mucosa yugal que corresponden a las manchas de Koplik, que son patognom\u00f3nicas del sarampi\u00f3n,"}}} {"id": 467, "year": 2020, "question_id_specific": 92, "full_question": "Mujer que presenta dificultad para caminar tras una cirug\u00eda ginecol\u00f3gica. Tiene dolor leve en el muslo y le falla la pierna al apoyar. En la exploraci\u00f3n presenta debilidad para la flexi\u00f3n de la cadera y para la extensi\u00f3n de la rodilla, y disestesias en la cara anterior del muslo. \u00bfCu\u00e1l es el diagn\u00f3stico de sospecha m\u00e1s probable?:", "full_answer": "El nervio femoral (aunque poco habitual) puede ser da\u00f1ado en una operaci\u00f3n ces\u00e1rea, una histerectom\u00eda o una operaci\u00f3n abdominal baja. Esta lesi\u00f3n nerviosa produce hipoestesia y debilidad a lo largo de su distribuci\u00f3n. La femoroneuropat\u00eda se manifiesta por una par\u00e1lisis (25% de los casos) de los m\u00fasculos cu\u00e1driceps, abolici\u00f3n de los reflejos rotulianos e hipoestesia de la cara anterior e interna del muslo. Por lo tanto, respuesta 1 correcta. El nervio f\u00e9moro cut\u00e1neo o cut\u00e1neo femoral lateral (L2,L3) es solo sensitivo encarg\u00e1ndose de la sensibilidad de la piel de la cara lateral y anterior del muslo hasta la rodilla, por lo que no explicar\u00eda el problema motor de la marcha y 2 es falsa. El nervio obturador (L2-L4) es un nervio mixto. A nivel motor inverva el obturador externo, pect\u00edneo, aproximador corto, los aproximadores (aductores) y gr\u00e1cil. A nivel sensitivo la c\u00e1psula de la articulaci\u00f3n coxofemoral, cara medial del muslo por encima de la rodilla. Por tanto ni afecta a la sensibilidad de la cara anterior del muslo ni a la extensi\u00f3n de la rodilla de modo que 3 es falsa. Una lesi\u00f3n del nervio ci\u00e1tico a nivel motor afecta a la flexi\u00f3n de la rodilla no a su extensi\u00f3n y a nivel distal a esta (no cuentan nada de cl\u00ednica distal) y a nivel sensitivo tambi\u00e9n es distal en la cara externa de la pierna y en el pie por lo que 4 tambi\u00e9n es falsa.", "type": "CIRUG\u00cdA ORTOP\u00c9DICA Y TRAUMATOLOG\u00cdA", "options": {"1": "Neuropat\u00eda del nervio femoral.", "2": "Meralgia parest\u00e9sica del nervio f\u00e9moro cut\u00e1neo.", "3": "Neuropat\u00eda del nervio obturador.", "4": "Neuropat\u00eda del nervio ci\u00e1tico.", "5": NaN}, "correct_option": 1, "explanations": {"1": {"exist": true, "char_ranges": [[0, 444]], "word_ranges": [[0, 69]], "text": "El nervio femoral (aunque poco habitual) puede ser da\u00f1ado en una operaci\u00f3n ces\u00e1rea, una histerectom\u00eda o una operaci\u00f3n abdominal baja. Esta lesi\u00f3n nerviosa produce hipoestesia y debilidad a lo largo de su distribuci\u00f3n. La femoroneuropat\u00eda se manifiesta por una par\u00e1lisis (25% de los casos) de los m\u00fasculos cu\u00e1driceps, abolici\u00f3n de los reflejos rotulianos e hipoestesia de la cara anterior e interna del muslo. Por lo tanto, respuesta 1 correcta."}, "2": {"exist": true, "char_ranges": [[445, 692]], "word_ranges": [[69, 114]], "text": "El nervio f\u00e9moro cut\u00e1neo o cut\u00e1neo femoral lateral (L2,L3) es solo sensitivo encarg\u00e1ndose de la sensibilidad de la piel de la cara lateral y anterior del muslo hasta la rodilla, por lo que no explicar\u00eda el problema motor de la marcha y 2 es falsa."}, "3": {"exist": true, "char_ranges": [[693, 1086]], "word_ranges": [[114, 181]], "text": "El nervio obturador (L2-L4) es un nervio mixto. A nivel motor inverva el obturador externo, pect\u00edneo, aproximador corto, los aproximadores (aductores) y gr\u00e1cil. A nivel sensitivo la c\u00e1psula de la articulaci\u00f3n coxofemoral, cara medial del muslo por encima de la rodilla. Por tanto ni afecta a la sensibilidad de la cara anterior del muslo ni a la extensi\u00f3n de la rodilla de modo que 3 es falsa."}, "4": {"exist": true, "char_ranges": [[1087, 1355]], "word_ranges": [[181, 237]], "text": "Una lesi\u00f3n del nervio ci\u00e1tico a nivel motor afecta a la flexi\u00f3n de la rodilla no a su extensi\u00f3n y a nivel distal a esta (no cuentan nada de cl\u00ednica distal) y a nivel sensitivo tambi\u00e9n es distal en la cara externa de la pierna y en el pie por lo que 4 tambi\u00e9n es falsa."}, "5": {"exist": false, "char_ranges": [], "word_ranges": [], "text": ""}}} {"id": 423, "year": 2018, "question_id_specific": 89, "full_question": "Una mujer de 55 a\u00f1os, diab\u00e9tica tipo 2 y obesa, en tratamiento con metformina, con Hb A1c de 8%, cuenta historia de infecciones urinarias de repetici\u00f3n. \u00bfCual de las siguientes opciones terap\u00e9uticas para asociar a la metformina considera la MENOS adecuada?", "full_answer": "Los efectos secundarios m\u00e1s frecuentes del uso de los inhibidores SGLT2 son las infecciones genitourinarias. Ya que nuestra paciente presenta ITUS de repetici\u00f3n, el f\u00e1rmaco menos indicado para el control de su diabetes ser\u00eda la opci\u00f3n 4.", "type": "ENDOCRINOLOG\u00cdA", "options": {"1": "Inhibidores DPP4.", "2": "Analogos GLP1.", "3": "Insulina basal.", "4": "Inhibidores SGLT2.", "5": NaN}, "correct_option": 4, "explanations": {"1": {"exist": false, "char_ranges": [], "word_ranges": [], "text": ""}, "2": {"exist": false, "char_ranges": [], "word_ranges": [], "text": ""}, "3": {"exist": false, "char_ranges": [], "word_ranges": [], "text": ""}, "4": {"exist": true, "char_ranges": [[0, 237]], "word_ranges": [[0, 37]], "text": "Los efectos secundarios m\u00e1s frecuentes del uso de los inhibidores SGLT2 son las infecciones genitourinarias. Ya que nuestra paciente presenta ITUS de repetici\u00f3n, el f\u00e1rmaco menos indicado para el control de su diabetes ser\u00eda la opci\u00f3n 4."}, "5": {"exist": false, "char_ranges": [], "word_ranges": [], "text": ""}}} {"id": 449, "year": 2018, "question_id_specific": 151, "full_question": "Un ni\u00f1o de 13 a\u00f1os presenta un cuadro subagudo de cefalea y diplopia. La exploraci\u00f3n neurol\u00f3gica muestra una par\u00e1lisis de la mirada vertical y la resonancia magn\u00e9tica una lesi\u00f3n captante de contraste en la regi\u00f3n pineal que obstruye el acueducto de Silvio. El diagn\u00f3stico m\u00e1s probable es:", "full_answer": "Los tumores germinales son los m\u00e1s frecuentes en ni\u00f1os en la regi\u00f3n pineal. Suelen captar homog\u00e9neamente y provocar hidrocefalia triventricular y Parinaud. Los glioblastomas suelen darse en adultos a nivel lobar, los meduloblastomas son los tumores malignos m\u00e1s frecuentes a nivel de fosa posterior en ni\u00f1os (vermis y techo del IV ventr\u00edculo) pero raros a nivel pineal y los meningiomas son raros en ni\u00f1os y tambi\u00e9n en la regi\u00f3n pineal.", "type": "NEUROCIRUG\u00cdA", "options": {"1": "Glioblastoma.", "2": "Meduloblastoma.", "3": "Tumor de c\u00e9lulas germinales.", "4": "Meningioma.", "5": NaN}, "correct_option": 3, "explanations": {"1": {"exist": true, "char_ranges": [[156, 211]], "word_ranges": [[22, 31]], "text": "Los glioblastomas suelen darse en adultos a nivel lobar,"}, "2": {"exist": true, "char_ranges": [[213, 368]], "word_ranges": [[31, 57]], "text": "los meduloblastomas son los tumores malignos m\u00e1s frecuentes a nivel de fosa posterior en ni\u00f1os (vermis y techo del IV ventr\u00edculo) pero raros a nivel pineal"}, "3": {"exist": true, "char_ranges": [[0, 155]], "word_ranges": [[0, 22]], "text": "Los tumores germinales son los m\u00e1s frecuentes en ni\u00f1os en la regi\u00f3n pineal. Suelen captar homog\u00e9neamente y provocar hidrocefalia triventricular y Parinaud."}, "4": {"exist": true, "char_ranges": [[371, 436]], "word_ranges": [[58, 70]], "text": "los meningiomas son raros en ni\u00f1os y tambi\u00e9n en la regi\u00f3n pineal."}, "5": {"exist": false, "char_ranges": [], "word_ranges": [], "text": ""}}} {"id": 433, "year": 2018, "question_id_specific": 118, "full_question": "Luis es un joven de 25 a\u00f1os que sufri\u00f3 esplenectom\u00eda tras accidente de bicicleta hace 1 a\u00f1o. Tiene un perro que Ie mordi\u00f3 hace 24 horas y le ha producido una peque\u00f1a herida en la mano derecha. Acude a su centro de salud (situada a 3 horas del hospital m\u00e1s cercano) por fiebre de 39\u00baC, dolor en la herida y malestar general. A la exploraci\u00f3n PA 100/60 mm Hg, FC 110 latidos por minuto, ligera inflamaci\u00f3n en la herida sin pus. \u00bfCual de las siguientes actuaciones est\u00e1 m\u00e1s indicada en este momento?", "full_answer": "Esta pregunta nos ha resultado complicada. La pregunta no tiene toda la informaci\u00f3n necesaria, pero usando lo que tenemos podemos intentar responder. El perro es dom\u00e9stico, conocido y puede ser sometido a vigilancia, por lo tanto habr\u00eda que esperar a que pase el periodo de observaci\u00f3n y no se recomienda profilaxis. En este punto solo tendremos que preocuparnos por la posible bacteriemia que puede estar ocurriendo en un paciente con posibilidad de alto riesgo por asplenia. El tratamiento de elecci\u00f3n en mordedura de perro es amoxicilina/clavul\u00e1nico pero no est\u00e1, as\u00ed que podemos usar otro como clindamicina m\u00e1s una cefalosporina o clindamicina + moxifloxacino pero de igual forma no est\u00e1 contemplado entre las alternativas. En un primer intento contestamos la 4, pero la respuesta correcta es la 3, por la segunda parte de la pregunta; es un paciente de alto riesgo con signos incipiente de sepsis por lo que debe de enviarse al hospital, aqu\u00ed la clave m\u00e1s que en el antibi\u00f3tico est\u00e1 en la postura a tomar, en la segunda parte de cada respuesta.. Se podr\u00eda impugnar ya que no dicen en cual comunidad ocurri\u00f3 la mordedura (si fuera en Ceuta o Melilla el protocolo puede variar).", "type": "ENFERMEDADES INFECCIOSAS Y MICROBIOLOG\u00cdA", "options": {"1": "Enviar al hospital para vacunar de rabia y t\u00e9tanos y mantener en observaci\u00f3n.", "2": "Limpiar la herida y administrar gamaglobulina inespec\u00edfica intramuscular.", "3": "Dar 400 mg de moxifloxacino oral y enviar al hospital.", "4": "Dar clindamicina 600 mg oral cada 8 horas y observaci\u00f3n.", "5": NaN}, "correct_option": 3, "explanations": {"1": {"exist": false, "char_ranges": [], "word_ranges": [], "text": ""}, "2": {"exist": false, "char_ranges": [], "word_ranges": [], "text": ""}, "3": {"exist": true, "char_ranges": [[728, 1049]], "word_ranges": [[113, 174]], "text": "En un primer intento contestamos la 4, pero la respuesta correcta es la 3, por la segunda parte de la pregunta; es un paciente de alto riesgo con signos incipiente de sepsis por lo que debe de enviarse al hospital, aqu\u00ed la clave m\u00e1s que en el antibi\u00f3tico est\u00e1 en la postura a tomar, en la segunda parte de cada respuesta.."}, "4": {"exist": true, "char_ranges": [[728, 1049]], "word_ranges": [[113, 174]], "text": "En un primer intento contestamos la 4, pero la respuesta correcta es la 3, por la segunda parte de la pregunta; es un paciente de alto riesgo con signos incipiente de sepsis por lo que debe de enviarse al hospital, aqu\u00ed la clave m\u00e1s que en el antibi\u00f3tico est\u00e1 en la postura a tomar, en la segunda parte de cada respuesta.."}, "5": {"exist": false, "char_ranges": [], "word_ranges": [], "text": ""}}} {"id": 337, "year": 2016, "question_id_specific": 31, "full_question": "Mujer de 20 a\u00f1os con tumoraci\u00f3n ov\u00e1rica de 15 cm, s\u00f3lido-qu\u00edstica, detectada por ecograf\u00eda tras presentar s\u00edntomas abdominales inespect\u00edficos. En el estudio histopatol\u00f3gico de la pieza correspondiente se encuentran dientes, pelos, zonas de epitelio intestinal, \u00e1reas de epitelio escamoso (15%) y bronquial, as\u00ed como elementos neuroectod\u00e9rmicos y embrionarios en varias de las preparaciones histol\u00f3gicas. En referencia a este caso, se\u00f1ale el diagn\u00f3stico correcto:", "full_answer": "La respuesta es la 3, teratoma qu\u00edstico maduro. Esto es as\u00ed por las caracter\u00edsticas propias del tumor:los teratomas maduros se hallan constituidos por tejidos adultos, procedentes de las tres hojas embrionarias, con neto predominio de los elementos ectod\u00e9rmicos (epidermis, estructuras piloseb\u00e1ceas y tejido neural). El teratoma maduro es un tumor de las c\u00e9lulas germinales ov\u00e1ricas.", "type": "GINECOLOG\u00cdA Y OBSTETRICIA", "options": {"1": "Teratocarcinoma.", "2": "Teratoma inmaduro.", "3": "Teratoma qu\u00edstico maduro.", "4": "Disgerminoma.", "5": NaN}, "correct_option": 3, "explanations": {"1": {"exist": false, "char_ranges": [], "word_ranges": [], "text": ""}, "2": {"exist": false, "char_ranges": [], "word_ranges": [], "text": ""}, "3": {"exist": true, "char_ranges": [[48, 316]], "word_ranges": [[8, 44]], "text": "Esto es as\u00ed por las caracter\u00edsticas propias del tumor:los teratomas maduros se hallan constituidos por tejidos adultos, procedentes de las tres hojas embrionarias, con neto predominio de los elementos ectod\u00e9rmicos (epidermis, estructuras piloseb\u00e1ceas y tejido neural)."}, "4": {"exist": false, "char_ranges": [], "word_ranges": [], "text": ""}, "5": {"exist": false, "char_ranges": [], "word_ranges": [], "text": ""}}} {"id": 296, "year": 2016, "question_id_specific": 17, "full_question": "Un hombre de 47 a\u00f1os, fumador activo, con historia de abuso de drogas por v\u00eda parenteral acude a Urgencias por un cuadro de 2 horas de evoluci\u00f3n de dolor costal izquierdo pleur\u00edtico de inicio agudo y disnea progresiva hasta hacerse de reposo. En las 2 semanas previas hab\u00eda presentado febr\u00edcula, malestar general y tos seca. A la exploraci\u00f3n el paciente impresiona de gravedad, est\u00e1 taquipneico a 36 rpm, con tiraje supraclavicular, saturaci\u00f3n de ox\u00edgeno por pulsioximetr\u00eda de 81%, cianosis, sudoraci\u00f3n, taquicardia a 135 latidos por minuto y PA de 75/47 mm Hg. La tr\u00e1quea est\u00e1 desviada hacia el lado derecho y la auscultaci\u00f3n demuestra ausencia completa de murmullo vesicular en hemit\u00f3rax izquierdo. Se realiza una serolog\u00eda VIH que resulta positiva y presenta un recuento de linfocitos CDA de 176 c\u00e9lulas/ml. \u00bfCu\u00e1l es el diagn\u00f3stico m\u00e1s probable?", "full_answer": "Paciente con factor de riesgo para adquisici\u00f3n de VIH, que acude a urgencias por un cuadro de inicio subagudo de 2 semanas de evoluci\u00f3n (febr\u00edcula, malestar general y tos seca) que se agrava de forma brusca (dolor pleur\u00edtico) y progresiva (disnea). El examen f\u00edsico sugiere un neumot\u00f3rax (taquipnea, tiraje, baja saturaci\u00f3n, cianosis, diaforesis, taquicardia, tr\u00e1quea desviada y ausencia de murmullo vesicular). Tiene una serolog\u00eda de para VIH positiva y un CD4 menos de 200 cel. /mL. La radiograf\u00eda muestra hiperclaridad en el campo pulmonar izquierdo con ausencia de vasos entre el l\u00edmite del pulm\u00f3n y la pared tor\u00e1cica del lado izquierdo, desplazamiento del mediastino y de la tr\u00e1quea al lado derecho, adem\u00e1s se observa un mu\u00f1\u00f3n en el hilio pulmonar izquierdo que corresponder\u00eda al pulm\u00f3n colapsado. La sospecha de VIH es alta y aunque est\u00e1 pendiente de confirmaci\u00f3n, en la urgencia es v\u00e1lido asumir esta opci\u00f3n. Dicho esto, busco patolog\u00edas que se asocien a este cuadro, la rotura de una bulla si bien se relaciona con la aparici\u00f3n de s\u00edntomas bruscos no va de la mano con los otros s\u00edntomas (hay que buscar algo que explique todo o casi todo), la neumon\u00eda si bien es cierto puede estar relacionada con buena parte de los s\u00edntomas no se correlaciona con la imagen, la TB pulmonar podr\u00eda estar manifestarse con todo lo antes dicho y provocar un neumot\u00f3rax, pero aqu\u00ed ante un cuadro respiratorio con m\u00e1s de 7 d\u00edas de evoluci\u00f3n que se complica de forma brusca, en un paciente con una alta sospecha de infecci\u00f3n por VIH la opci\u00f3n m\u00e1s clara va por la neumon\u00eda por P. jirovecii. En los pacientes con Pneumocystis, el 10 %, aparecen neumatoceles que en ocasiones pueden abrirse al espacio pleural produciendo un neumot\u00f3rax espont\u00e1neo. Recordando, los g\u00e9rmenes m\u00e1s frecuentes en la afectaci\u00f3n pulmonar en pacientes con infecci\u00f3n por VIH/SIDA son P. jiroveci (solo o asociado a tuberculosis u otras micobacterias), S. pneumoniae, H. influenzae, S. aureus, citomegalovirus, Legionella, estafilococos y bacilos gramnegativos (Pseudomonas). Los pacientes con neumon\u00eda por P. jirovecii presentan, en el 90% de casos, un recuento de CD4< 250 cel. /mL. Otros agentes que pueden provocar afecci\u00f3n pulmonar con menor frecuencia son Rhodococcus equi, Nocardia, Aspergillus, Cryptococcus, Toxoplasma yLeishmania. Aunque la frecuencia de las diferentes infecciones oportunistas ha variado como consecuencia de la administraci\u00f3n de los tratamientos antirretrovirales de alta eficacia, nuestro paciente estar\u00eda en un escenario donde a\u00fan no recibi\u00f3 TARGA.", "type": "ENFERMEDADES INFECCIOSAS", "options": {"1": "Rotura de bulla enfisematosa subpleural.", "2": "Neumon\u00eda bacteriana necrotizante adquirida en la comunidad.", "3": "Tuberculosis pulmonar.", "4": "Neumon\u00eda por Pneumocystis jiroveci.", "5": NaN}, "correct_option": 4, "explanations": {"1": {"exist": true, "char_ranges": [[975, 1147]], "word_ranges": [[156, 190]], "text": "la rotura de una bulla si bien se relaciona con la aparici\u00f3n de s\u00edntomas bruscos no va de la mano con los otros s\u00edntomas (hay que buscar algo que explique todo o casi todo),"}, "2": {"exist": true, "char_ranges": [[1149, 1267]], "word_ranges": [[190, 211]], "text": "la neumon\u00eda si bien es cierto puede estar relacionada con buena parte de los s\u00edntomas no se correlaciona con la imagen,"}, "3": {"exist": true, "char_ranges": [[1375, 1576]], "word_ranges": [[229, 267]], "text": "un cuadro respiratorio con m\u00e1s de 7 d\u00edas de evoluci\u00f3n que se complica de forma brusca, en un paciente con una alta sospecha de infecci\u00f3n por VIH la opci\u00f3n m\u00e1s clara va por la neumon\u00eda por P. jirovecii."}, "4": {"exist": true, "char_ranges": [[1375, 1576]], "word_ranges": [[229, 267]], "text": "un cuadro respiratorio con m\u00e1s de 7 d\u00edas de evoluci\u00f3n que se complica de forma brusca, en un paciente con una alta sospecha de infecci\u00f3n por VIH la opci\u00f3n m\u00e1s clara va por la neumon\u00eda por P. jirovecii."}, "5": {"exist": false, "char_ranges": [], "word_ranges": [], "text": ""}}} {"id": 76, "year": 2012, "question_id_specific": 87, "full_question": "Mujer de 76 a\u00f1os con obesidad y diabetes mellitas tipo 2 en tratamiento con metformina. Acude a urgencias porque desde hace 3 d\u00edas presenta fiebre de 38\u00baC, poliaquiuria, disuria, intensa sed y disminuci\u00f3n progresiva del nivel de consciencia. En la exploraci\u00f3n f\u00edsica se aprecian signos de deshidrataci\u00f3n, PA de 95/54 mm Hg y no hay signos de localidad neurol\u00f3gica. En la anal\u00edtica destaca leucocitosis, creatinina de 1,8 mg/dL y glucemia de 855 mg/dL. \u00bfCu\u00e1l es el diagn\u00f3stico mas probable?", "full_answer": "La pregunta cuenta el caso de una paciente diab\u00e9tica tipo 2 con hiperglucemia y disminuci\u00f3n del nivel de conciencia sin focalidad neurol\u00f3gica en el contexto de una infecci\u00f3n urinaria y que ha desarrollado una insuficiencia renal aguda de probable origen prerrenal. La opci\u00f3n 2 se descarta de forma inmediata puesto que el cuadro cl\u00ednico no es de insuficiencia cardiaca. La opci\u00f3n 3 tambi\u00e9n se descarta porque no cuadra con la cl\u00ednica y adem\u00e1s el HSD dar\u00eda cl\u00ednica neurol\u00f3gica con localidad. La opci\u00f3n 4 se descarta puesto que la insuficiencia renal obstructiva no da poliaquiuria, disuria ni el resto de la cl\u00ednica. La opci\u00f3n 5 es menos probable porque la cetoacidosis diab\u00e9tica es mas tipica de DM tipo 1. Por tanto la opci\u00f3n correcta es la 1.", "type": "ANESTESIOLOG\u00cdA, CUIDADOS CR\u00cdTICOS Y URGENCIAS", "options": {"1": "Coma hipergluc\u00e9mico hiperosmolar no cet\u00f3sico.", "2": "Insuficiencia cardiaca.", "3": "Hematoma subdural.", "4": "Insuficiencia renal de causa obstructiva.", "5": "Cetoacidosis diab\u00e9tica con coma."}, "correct_option": 1, "explanations": {"1": {"exist": false, "char_ranges": [], "word_ranges": [], "text": ""}, "2": {"exist": true, "char_ranges": [[265, 369]], "word_ranges": [[41, 59]], "text": "La opci\u00f3n 2 se descarta de forma inmediata puesto que el cuadro cl\u00ednico no es de insuficiencia cardiaca."}, "3": {"exist": true, "char_ranges": [[370, 490]], "word_ranges": [[59, 80]], "text": "La opci\u00f3n 3 tambi\u00e9n se descarta porque no cuadra con la cl\u00ednica y adem\u00e1s el HSD dar\u00eda cl\u00ednica neurol\u00f3gica con localidad."}, "4": {"exist": true, "char_ranges": [[491, 615]], "word_ranges": [[80, 101]], "text": "La opci\u00f3n 4 se descarta puesto que la insuficiencia renal obstructiva no da poliaquiuria, disuria ni el resto de la cl\u00ednica."}, "5": {"exist": true, "char_ranges": [[616, 706]], "word_ranges": [[101, 118]], "text": "La opci\u00f3n 5 es menos probable porque la cetoacidosis diab\u00e9tica es mas tipica de DM tipo 1."}}} {"id": 159, "year": 2012, "question_id_specific": 111, "full_question": "Un paciente var\u00f3n de 45 a\u00f1os de edad con antecedentes de litiasis \u00farica con c\u00f3licos nefr\u00edticos expulsivos de repetici\u00f3n desde hace 25 a\u00f1os, acude a Urgencias refiriendo palpitaciones y dolor lumbar derecho intenso desde hace 2 horas antes. El an\u00e1lisis de orina muestra un pH de 5,5 y leucocituria sin proteinuria. El ECG confirma una fibrilaci\u00f3n auricular no conocidad previamente. En la bioqu\u00edmica sangu\u00ednea destaca una creatinina de 0.9mg/dl, un calcio de 11mg/dl y una LDH de 950 U/l. \u00bfCual es la maniobra diagn\u00f3stica mas \u00fatil a realizar?", "full_answer": "En esta pregunta tengo mas dudas en cuanto a cual es la respuesta correcta, pues debemos interpretar cual es la prueba mas \u00fatil, con cual podemos obtener mas rendimiento. No obstante esta situaci\u00f3n depende as\u00ed mismo de los servicios de radiolog\u00eda de cada centro y de sus preferencias. Me centrar\u00e9 en la parte urol\u00f3gica, dejando de lado el resto de sintomatolog\u00eda. \u2022 La Rx de abdomen es el primer estudio de imagen a realizar por ser r\u00e1pido, sencillo y pr\u00e1cticamente inocuo (salvo en ni\u00f1os y gestantes). El 90% de los c\u00e1lculos son radiopacos y por lo tanto visibles, pero la sensibilidad de esta prueba, utilizada de manera aislada, disminuye hasta el 45-59%. Es una prueba obligatoria en toda sospecha de dolor c\u00f3lico. No obstante con menor rendimiento que las otras. Aunque la historia urol\u00f3gica hable de litiasis \u00farica (radiotrasparente), con un ph \u00e1cido que reafirma el diagn\u00f3stico de origen \u00farico, no podemos descartar nunca la litiasis de oxalato c\u00e1lcico que puede aparecer de forma sincr\u00f3nica. \u2022 La UIV me parece una de las opciones con mayor rendimiento. Nos dar\u00e1 claridad diagn\u00f3stica de obstrucci\u00f3n, de su magnitud y de su repercusi\u00f3n renal. Considerada durante mucho tiempo la exploraci\u00f3n de elecci\u00f3n para el estudio de los pacientes con litiasis renal. Es una prueba con sensibilidad y especificidad elevadas (87-90% y 94-100%, respectivamente), relativamente inocua, y al alcance de todos los hospitales. Nos informa del n\u00famero, tama\u00f1o, forma y situaci\u00f3n del c\u00e1lculo, as\u00ed como del estado funcional del ri\u00f1\u00f3n afecto. \u2022 El TAC con contraste me parece la opci\u00f3n con mayor rendimiento. Sirve para descubrir litiasis que otras pruebas no logran encontrar y nos da una imagen global del resto de estructuras. Gran rendimiento pero tambi\u00e9n mas cara, mas tiempo, mas radiaci\u00f3n. Se ha confirmado como una exploraci\u00f3n de gran sensibilidad y especificidad, superando a las anteriores pruebas (94-100% y 92-100%, respectivamente, para la detecci\u00f3n de c\u00e1lculo ureteral) y con sus numerosas ventajas se ha convertido en el examen de referencia por lo que se espera que reemplazar\u00e1 a UIV y la ecograf\u00eda en todo el mundo. Realiza un barrido helicoidal con cortes de 5 mm que permite la detecci\u00f3n de c\u00e1lculos de hasta 2 mm. Con la administraci\u00f3n de contraste, permite la evaluaci\u00f3n de la funci\u00f3n renal. \u2022 La Ecograf\u00eda es una prueba r\u00e1pida, barata e inocua, pero muy radi\u00f3log-dependiente. Da gran informaci\u00f3n renal sobre repercusi\u00f3n obstructiva, pero menor calidad de cara a encontrar una litiasis. La ecograf\u00eda s\u00f3lo detecta c\u00e1lculos mayores de 4 mm, situados en la uni\u00f3n pielo-ureteral o la uni\u00f3n ur\u00e9tero-vesical, mientras que el ur\u00e9ter lumbar y pelviano es poco accesible debido a la interposici\u00f3n de las asas intestinales. Por ello, la ecograf\u00eda realizada de manera aislada es poco sensible (20-45%) para la detecci\u00f3n de c\u00e1lculos, pero asociada a la radiograf\u00eda simple de abdomen aumenta su sensibilidad y especificidad. \u2022 La ecocardiograf\u00eda doppler no tiene uso en el diagn\u00f3stico urol\u00f3gico. Mi respuesta ser\u00eda la TAC con contraste ev y fase excretora (respuesta (c).), en cuanto a rendimiento, sensibilidad y especificidad. Hay servicio de radiolog\u00eda mas h\u00e1biles o predispuestos a una u otra. Sin olvidar que la Rx de abdomen siempre debe hacerse en la sospecha de un c\u00f3lico.", "type": "UROLOG\u00cdA", "options": {"1": "Radiograf\u00eda simple de abdomen.", "2": "UIV.", "3": "TAC con contraste.", "4": "ECOgraf\u00eda abdominal.", "5": "ECOCARDIOGRAF\u00cdA doppler."}, "correct_option": 3, "explanations": {"1": {"exist": false, "char_ranges": [], "word_ranges": [], "text": ""}, "2": {"exist": true, "char_ranges": [[1002, 1262]], "word_ranges": [[166, 208]], "text": "La UIV me parece una de las opciones con mayor rendimiento. Nos dar\u00e1 claridad diagn\u00f3stica de obstrucci\u00f3n, de su magnitud y de su repercusi\u00f3n renal. Considerada durante mucho tiempo la exploraci\u00f3n de elecci\u00f3n para el estudio de los pacientes con litiasis renal."}, "3": {"exist": true, "char_ranges": [[1529, 1713]], "word_ranges": [[248, 279]], "text": "El TAC con contraste me parece la opci\u00f3n con mayor rendimiento. Sirve para descubrir litiasis que otras pruebas no logran encontrar y nos da una imagen global del resto de estructuras."}, "4": {"exist": true, "char_ranges": [[2299, 2491]], "word_ranges": [[376, 404]], "text": "La Ecograf\u00eda es una prueba r\u00e1pida, barata e inocua, pero muy radi\u00f3log-dependiente. Da gran informaci\u00f3n renal sobre repercusi\u00f3n obstructiva, pero menor calidad de cara a encontrar una litiasis."}, "5": {"exist": true, "char_ranges": [[2919, 2987]], "word_ranges": [[471, 481]], "text": "La ecocardiograf\u00eda doppler no tiene uso en el diagn\u00f3stico urol\u00f3gico."}}} {"id": 52, "year": 2011, "question_id_specific": 144, "full_question": "Un paciente de 36 a\u00f1os, oriundo de otra cultura que vive en Espa\u00f1a desde hace 4 a\u00f1os se presenta en una consulta de S.N. de Salud. Refiere presentar desde hace 10 meses sintomatolog\u00eda ansiosa y humor depresivo. Este cuadro interfiere moderadamente en su actividad cotidiana. No tiene antecedentes psiqui\u00e1tricos previos. Dicha situaci\u00f3n se produce a ra\u00edz del fallecimiento, en un accidente automovil\u00edstico, de un hermano mayor con quien se encontraba muy unido. Se\u00f1alar cu\u00e1l de los siguientes diagn\u00f3sticos es el apropiado:", "full_answer": "La relativa levedad de los s\u00edntomas, que no interfieren de forma grave en su actividad diaria, excluye en principio el diagn\u00f3stico de depresi\u00f3n mayor. No hay datos de fases hipoman\u00edacas que justifiquen un trastorno bipolar II. La distimia precisa una duraci\u00f3n de al menos dos a\u00f1os. La ciclotimia es un diagn\u00f3stico hoy en d\u00eda muy raro (porque, como dec\u00eda Castilla del Pino en una entrevista, lo que antes eran ciclot\u00edmicos los llamamos ahora bipolares, contribuyendo a una de las m\u00faltiples epidemias psiqui\u00e1tricas de nuestro tiempo), que consta de oscilaciones entre el polo depresivo y el maniforme, leves. Pero no hay datos de tales oscilaciones. La respuesta correcta es, con toda probabilidad, la 3. El trastorno adaptativo aparece en los 3 meses posteriores al acontecimiento estresante y no dura m\u00e1s de 6 meses si ha cesado el mismo o sus consecuencias (lo que no es el caso porque el hermano del imaginario paciente sigue muerto). Los s\u00edntomas ansioso-depresivos leves encajan con tal diagn\u00f3stico. Una muestra m\u00e1s de c\u00f3mo conceptualizar como enfermedad lo que son hechos (aunque dolorosos) de la vida. Seguro que el imaginario paciente se lleva alg\u00fan antidepresivo porque a lo mejor algo ayuda (1).", "type": "PSIQUIATR\u00cdA", "options": {"1": "Depresi\u00f3n mayor.", "2": "Trastorno bipolar II.", "3": "Trastorno de adaptaci\u00f3n.", "4": "Distimia.", "5": "Ciclotimia."}, "correct_option": 3, "explanations": {"1": {"exist": true, "char_ranges": [[0, 150]], "word_ranges": [[0, 24]], "text": "La relativa levedad de los s\u00edntomas, que no interfieren de forma grave en su actividad diaria, excluye en principio el diagn\u00f3stico de depresi\u00f3n mayor."}, "2": {"exist": true, "char_ranges": [[151, 226]], "word_ranges": [[24, 36]], "text": "No hay datos de fases hipoman\u00edacas que justifiquen un trastorno bipolar II."}, "3": {"exist": true, "char_ranges": [[703, 1003]], "word_ranges": [[113, 162]], "text": "El trastorno adaptativo aparece en los 3 meses posteriores al acontecimiento estresante y no dura m\u00e1s de 6 meses si ha cesado el mismo o sus consecuencias (lo que no es el caso porque el hermano del imaginario paciente sigue muerto). Los s\u00edntomas ansioso-depresivos leves encajan con tal diagn\u00f3stico."}, "4": {"exist": true, "char_ranges": [[227, 281]], "word_ranges": [[36, 46]], "text": "La distimia precisa una duraci\u00f3n de al menos dos a\u00f1os."}, "5": {"exist": true, "char_ranges": [[282, 647]], "word_ranges": [[46, 104]], "text": "La ciclotimia es un diagn\u00f3stico hoy en d\u00eda muy raro (porque, como dec\u00eda Castilla del Pino en una entrevista, lo que antes eran ciclot\u00edmicos los llamamos ahora bipolares, contribuyendo a una de las m\u00faltiples epidemias psiqui\u00e1tricas de nuestro tiempo), que consta de oscilaciones entre el polo depresivo y el maniforme, leves. Pero no hay datos de tales oscilaciones."}}} {"id": 297, "year": 2016, "question_id_specific": 50, "full_question": "Un hombre de 58 a\u00f1os refer\u00eda una historia de semanas de evoluci\u00f3n con lesiones cut\u00e1neas progresivas, medianamente dolorosas, en su brazo izquierdo. Hab\u00eda empezado como una lesi\u00f3n eritematosa en su pulgar izquierdo. Ten\u00eda unas estr\u00edas rojizas visibles como l\u00edneas de conexi\u00f3n entre las lesiones. El paciente no presentaba fiebre ni otros s\u00edntomas generales. Hab\u00eda estado trabajando en su jard\u00edn pero no recordaba haberse hecho ninguna herida. El diagn\u00f3stico etiol\u00f3gico se realiz\u00f3 por cultivo de una biopsia cut\u00e1nea. \u00bfCu\u00e1l es el agente causal m\u00e1s probable de este proceso?", "full_answer": "El paciente desarrolla una actividad en contacto con tierra y plantas (que nos la proporcionan de una forma no gratuita). Los s\u00edntomas son cr\u00f3nicos, que empezaron de forma distal (pulgar izquierdo) y van afectando al brazo ipsilateral (estr\u00edas rojizas). No hab\u00eda s\u00edntomas sist\u00e9micos. Comentan adem\u00e1s que el diagn\u00f3stico fue hecho a partir de una biopsia cut\u00e1nea. De los microorganismos comentados, el Microsporum gypseum no suele manifestar la diseminaci\u00f3n de la infecci\u00f3n de un aspecto linfang\u00edtico, en general los hongos queratin\u00f3filos (dermatofitos)no tienen ninguna capacidad de invadir tejidos profundos, as\u00ed que descartamos esta opci\u00f3n. La infecci\u00f3n por Staphylococcus aureus puede afectar capas superficiales y profundas de los tejidos, se pueden diseminar siguiendo un trayecto linfang\u00edtico, pero no es usual una evoluci\u00f3n tan larga, suele ser aguda (d\u00edas) y acompa\u00f1ada de s\u00edntomas sist\u00e9micos por lo general, por eso descartamos esta opci\u00f3n. El Mycobacterium marinum (y no Mycobacterum marinum) es una micobacteria no tuberculosa que se caracteriza por ser ambiental, oportunista y fotocrom\u00f3gena. Tiene un crecimiento lento, de 2 a 8 semanas, a una temperatura que oscila entre los 30 a 37 \u00b0C. Afecta a diferentes especies de peces tanto de aguas fr\u00edas, c\u00e1lidas, dulces o saladas, especialmente aguas estancadas de peceras y piscinas sin cloro. La infecci\u00f3n en el humano se produce por el contacto directo con peces o con aguas contaminadas en presencia de una p\u00e9rdida de continuidad en la piel del hu\u00e9sped. Nuestro paciente no tiene antecedente de exposici\u00f3n a peces o agua (no lo dicen), as\u00ed que teniendo en cuenta que por lo general el diagn\u00f3stico se basa en un alto \u00edndice de sospecha y preguntando de forma espec\u00edfica este antecedente, nuestro caso se aleja de esta posibilidad. El complejo Sporothrix schenckii es el agente causal de la Esporotricosis, este hongo dim\u00f3rfico se ubica en el suelo, el complejo est\u00e1 formado por 5 especies, S. schenckii sensu strictu, S. brasiliensis y S. globosa, capaces de causar enfermedad humana y animal; las otras dos son S. mexicana y S. albicana no asociadas con ninguna enfermedad. Los pacientes se suelen contagiar por la exposici\u00f3n a la tierra mediante un objeto punzante (espinas, tallos, garras de animales, etc) y presentan una evoluci\u00f3n lenta, afecta tejidos profundos y su diseminaci\u00f3n t\u00edpicamente suele seguir un trayecto como el descrito en la pregunta.", "type": "ENFERMEDADES INFECCIOSAS", "options": {"1": "Dermatofitosis por Microsporum gypseum.", "2": "Infecci\u00f3n cut\u00e1nea por Staphylococcus aureus.", "3": "Esporotricosis.", "4": "Infecci\u00f3n cut\u00e1nea por Mycobacterium marinum.", "5": NaN}, "correct_option": 3, "explanations": {"1": {"exist": true, "char_ranges": [[397, 607]], "word_ranges": [[60, 88]], "text": "el Microsporum gypseum no suele manifestar la diseminaci\u00f3n de la infecci\u00f3n de un aspecto linfang\u00edtico, en general los hongos queratin\u00f3filos (dermatofitos)no tienen ninguna capacidad de invadir tejidos profundos,"}, "2": {"exist": true, "char_ranges": [[642, 914]], "word_ranges": [[93, 134]], "text": "La infecci\u00f3n por Staphylococcus aureus puede afectar capas superficiales y profundas de los tejidos, se pueden diseminar siguiendo un trayecto linfang\u00edtico, pero no es usual una evoluci\u00f3n tan larga, suele ser aguda (d\u00edas) y acompa\u00f1ada de s\u00edntomas sist\u00e9micos por lo general,"}, "3": {"exist": true, "char_ranges": [[2135, 2415]], "word_ranges": [[335, 378]], "text": "Los pacientes se suelen contagiar por la exposici\u00f3n a la tierra mediante un objeto punzante (espinas, tallos, garras de animales, etc) y presentan una evoluci\u00f3n lenta, afecta tejidos profundos y su diseminaci\u00f3n t\u00edpicamente suele seguir un trayecto como el descrito en la pregunta."}, "4": {"exist": true, "char_ranges": [[1515, 1790]], "word_ranges": [[232, 279]], "text": "Nuestro paciente no tiene antecedente de exposici\u00f3n a peces o agua (no lo dicen), as\u00ed que teniendo en cuenta que por lo general el diagn\u00f3stico se basa en un alto \u00edndice de sospecha y preguntando de forma espec\u00edfica este antecedente, nuestro caso se aleja de esta posibilidad."}, "5": {"exist": false, "char_ranges": [], "word_ranges": [], "text": ""}}} {"id": 74, "year": 2012, "question_id_specific": 63, "full_question": "Una mujer de 67 a\u00f1os con antecedentes de insuficiencia cardiaca congestiva (que lleva tratamiento con ramipril y furosemida) y diabetes tipo 2 (en tratamiento con insulina) acue a urgencias por disnea. En gasometr\u00eda realizada respirando aire ambiente se observa un pH 7.45, paO2 56, PaCO2 30 mm Hg, HCO3 26 mm Hg. \u00bfQu\u00e9 alteraci\u00f3n gasom\u00e9trica presenta la paciente?", "full_answer": "Esta pregunta me ha parecido complicadilla. Los hallazgos por separado son: pH normal pero con tendencia alcalosis, Insuficencia respiratoria aguda, hipocapnia y bicarbonato normal. La respuesta mas apropiada me parece la 1, dado que la paciente no presenta una alcalosis metab\u00f3lica (el bicarbonato estar\u00eda mas elevado), mucho menos una acidosis (el bicarbonato estar\u00eda bajo). La alcalosis tampoco es mixta, sino que es principalmente respiratoria. Probablemente la paciente est\u00e9 hiperventilando por su insuficiencia respiratoria, y el ri\u00f1\u00f3n que siempre compensa mas tarde est\u00e1 empezando a retener HCO3 para compensar y mantener el pH. Por tanto creo que la respuesta correcta es la 1.", "type": "ANESTESIOLOG\u00cdA, CUIDADOS CR\u00cdTICOS Y URGENCIAS", "options": {"1": "Insuficiencia respiratoria hipox\u00e9mica con alcalosis respiratoria compensada.", "2": "Alcalosis metab\u00f3lica aguda con insuficiencia respiratoria hipox\u00e9mica e hipoc\u00e1pnica.", "3": "Insuficiencia respiratoria normocapnica cr\u00f3nica, sin alteraci\u00f3n del equilibrio acido-base.", "4": "Insuficiencia respiratoria hipox\u00e9mica cr\u00f3nica con acidosis metab\u00f3lica compensada.", "5": "Insuficiencia respiratoria hipox\u00e9mica con alcalosis mixta compensada."}, "correct_option": 1, "explanations": {"1": {"exist": true, "char_ranges": [[182, 635]], "word_ranges": [[24, 92]], "text": "La respuesta mas apropiada me parece la 1, dado que la paciente no presenta una alcalosis metab\u00f3lica (el bicarbonato estar\u00eda mas elevado), mucho menos una acidosis (el bicarbonato estar\u00eda bajo). La alcalosis tampoco es mixta, sino que es principalmente respiratoria. Probablemente la paciente est\u00e9 hiperventilando por su insuficiencia respiratoria, y el ri\u00f1\u00f3n que siempre compensa mas tarde est\u00e1 empezando a retener HCO3 para compensar y mantener el pH."}, "2": {"exist": false, "char_ranges": [], "word_ranges": [], "text": ""}, "3": {"exist": false, "char_ranges": [], "word_ranges": [], "text": ""}, "4": {"exist": false, "char_ranges": [], "word_ranges": [], "text": ""}, "5": {"exist": false, "char_ranges": [], "word_ranges": [], "text": ""}}} {"id": 288, "year": 2016, "question_id_specific": 227, "full_question": "Paciente de 76 a\u00f1os, con hipertensi\u00f3n y diabetes mellitus, que acude a Urgencias porque desde hace 72 horas ha comenzado con un cuadro de palpitaciones y disminuci\u00f3n de su capacidad para hacer esfuerzos. A su llegada se documenta una fibrilaci\u00f3n auricular con respuesta ventricular en torno a 120 lpm. \u00bfCu\u00e1l de las siguientes opciones es FALSA?:", "full_answer": "Al ser el primer episodio de fibrilaci\u00f3n auricular es el candidato id\u00f3neo para realizar una ablaci\u00f3n con cat\u00e9ter.", "type": "CARDIOLOG\u00cdA Y CIRUG\u00cdA VASCULAR", "options": {"1": "Este paciente debe estar anticoagulado oralmente de por vida, salvo contraindicaci\u00f3n.", "2": "Si decidimos realizar una cardioversi\u00f3n a su llegada a Urgencias, ser\u00eda necesario hacer previamente una ecocardiograf\u00eda transesof\u00e1gica.", "3": "Al ser el primer episodio de fibrilaci\u00f3n auricular es el candidato id\u00f3neo para realizar una ablaci\u00f3n con cat\u00e9ter.", "4": "Para frenar la frecuencia cardiaca podr\u00edamos emplear betabloqueantes.", "5": NaN}, "correct_option": 3, "explanations": {"1": {"exist": false, "char_ranges": [], "word_ranges": [], "text": ""}, "2": {"exist": false, "char_ranges": [], "word_ranges": [], "text": ""}, "3": {"exist": true, "char_ranges": [[0, 113]], "word_ranges": [[0, 18]], "text": "Al ser el primer episodio de fibrilaci\u00f3n auricular es el candidato id\u00f3neo para realizar una ablaci\u00f3n con cat\u00e9ter."}, "4": {"exist": false, "char_ranges": [], "word_ranges": [], "text": ""}, "5": {"exist": false, "char_ranges": [], "word_ranges": [], "text": ""}}} {"id": 460, "year": 2018, "question_id_specific": 174, "full_question": "Mujer de 60 a\u00f1os, diab\u00e9tica insulinodependiente, que presenta omalgia derecha de predominio nocturno de varias semanas de evoluci\u00f3n. No refiere traumatismo. En la exploraci\u00f3n f\u00edsica destaca una limitaci\u00f3n activa y pasiva de todos los arcos de movimiento del hombro. \u00bfQu\u00e9 cuadro cl\u00ednico sospecha como primera posibilidad diagn\u00f3stica?", "full_answer": "Todo apunta a una capsulitis adhesiva, como bien dices. La edad, el sexo, la cl\u00ednica\u2026 lo de la DM\u2026", "type": "TRAUMATOLOG\u00cdA Y ORTOPED\u00cdA", "options": {"1": "Una tumoraci\u00f3n maligna localizada en la ep\u00edfisis proximal de h\u00famero.", "2": "Una artritis s\u00e9ptica de hombro.", "3": "Una capsulitis adhesiva.", "4": "Un proceso degenerativo artr\u00f3sico acromioclavicular.", "5": NaN}, "correct_option": 3, "explanations": {"1": {"exist": false, "char_ranges": [], "word_ranges": [], "text": ""}, "2": {"exist": false, "char_ranges": [], "word_ranges": [], "text": ""}, "3": {"exist": true, "char_ranges": [[0, 98]], "word_ranges": [[0, 19]], "text": "Todo apunta a una capsulitis adhesiva, como bien dices. La edad, el sexo, la cl\u00ednica\u2026 lo de la DM\u2026"}, "4": {"exist": false, "char_ranges": [], "word_ranges": [], "text": ""}, "5": {"exist": false, "char_ranges": [], "word_ranges": [], "text": ""}}} {"id": 333, "year": 2016, "question_id_specific": 90, "full_question": "Una mujer de 35 a\u00f1os consulta por un n\u00f3dulo tiroideo derecho detectado incidentalmente una ma\u00f1ana al observarse en el espejo un bulto en la cara anterior del cuello. Tras los estudios pertinentes se decide intervenir quir\u00fargicamente a la paciente realiz\u00e1ndose una tiroidectomia total con vaciamiento del componente ganglionar central, siendo el informe del pat\u00f3logo de n\u00f3dulo tiroideo de 2,3 cm ocupado en su totalidad por un carcinoma papilar de tiroides, variante c\u00e9lulas altas, sin infiltraci\u00f3n vascular pero s\u00ed capsular. Las concentraciones de tiroglobulina 24 h despu\u00e9s de la tiroidectomia son de 14 ng/mL. \u00bfCu\u00e1l ser\u00eda el siguiente paso que usted dar\u00eda en esta paciente?", "full_answer": "La comprobaci\u00f3n de la presencia de tiroglobulina en sangre y el desarrollo de t\u00e9cnicas mediante las cuales es posible determinarla, han permitido contar con un nuevo procedimiento para el control de pacientes con carcinoma papilar o folicular del tiroides. La tiroglobulina es sintetizada en las c\u00e9lulas foliculares y luego secretada a la circulaci\u00f3n, por lo tanto, despu\u00e9s de la tiroidectom\u00eda total debe desaparecer, si eso no ocurre hay que pensar en la existencia de restos de tejido de met\u00e1stasis.", "type": "ENDOCRINOLOG\u00cdA", "options": {"1": "Remitir de nuevo a la paciente al cirujano para que realice un vaciamiento ganglionar laterocervical derecho.", "2": "Iniciar tratamiento con una dosis de levotiroxina supresora de TSH y citarla a revisi\u00f3n 6 meses despu\u00e9s con una nueva determinaci\u00f3n anal\u00edtica de tiroglobulina y una ecograf\u00eda cervical.", "3": "Solicitar un PET-TAC para ver si hay afectaci\u00f3n ganglionar.", "4": "Demorar el inicio del tratamiento sustitutivo-supresor de TSH con levotiroxina y remitir la paciente al servicio de Medicina Nuclear para la administraci\u00f3n de una dosis ablativa de 100 mCi de I131.", "5": NaN}, "correct_option": 4, "explanations": {"1": {"exist": false, "char_ranges": [], "word_ranges": [], "text": ""}, "2": {"exist": false, "char_ranges": [], "word_ranges": [], "text": ""}, "3": {"exist": false, "char_ranges": [], "word_ranges": [], "text": ""}, "4": {"exist": true, "char_ranges": [[257, 501]], "word_ranges": [[39, 79]], "text": "La tiroglobulina es sintetizada en las c\u00e9lulas foliculares y luego secretada a la circulaci\u00f3n, por lo tanto, despu\u00e9s de la tiroidectom\u00eda total debe desaparecer, si eso no ocurre hay que pensar en la existencia de restos de tejido de met\u00e1stasis."}, "5": {"exist": false, "char_ranges": [], "word_ranges": [], "text": ""}}} {"id": 185, "year": 2013, "question_id_specific": 66, "full_question": "Un paciente de 74 a\u00f1os con resecci\u00f3n intestinal tras una trombosis mesent\u00e9rica, acude a consulta por cansancio y dolor \u00f3seo. Hab\u00eda sufrido una fractura de Colles tres meses antes. Los estudios anal\u00edticos ponen de manifiesto una anemia de 9.5 g/dl, microc\u00edtica, alb\u00famina 3.5 g/di, calcio 7.5 mg/dl, fosfato 2.0 mg/dl y fosfatasa alcalina 224 UI (normal hasta 120 UI). Las radiograf\u00edas del f\u00e9mur muestran unas bandas radiol\u00facidas perpendiculares a la cortical en el lado interno de su parte superior. \u00bfCu\u00e1l es el diagn\u00f3stico m\u00e1s probable?:", "full_answer": "Aunque esta no me la hab\u00edan asignado, la respondo tambi\u00e9n ya que trata un tema de endocrinolog\u00eda: la vitamina D. En este caso hay datos importantes como el antecedente de resecci\u00f3n intestinal, la hipocalcemia y ha hipofosforemoia; esto nos debe hacer pensar en un d\u00e9ficit de vitamina D por malabsorci\u00f3n y por tanto, en una osteomalacia. El resto de datos: fractura previa, dolores y la radiograf\u00eda apoyan el diagn\u00f3stico.", "type": "ENDOCRINOLOG\u00cdA", "options": {"1": "Osteoporosis.", "2": "Enfermedad de Paget.", "3": "Osteitis fibrosa.", "4": "Met\u00e1stasis de c\u00e1ncer de pr\u00f3stata.", "5": "Osteomalacia."}, "correct_option": 5, "explanations": {"1": {"exist": false, "char_ranges": [], "word_ranges": [], "text": ""}, "2": {"exist": false, "char_ranges": [], "word_ranges": [], "text": ""}, "3": {"exist": false, "char_ranges": [], "word_ranges": [], "text": ""}, "4": {"exist": false, "char_ranges": [], "word_ranges": [], "text": ""}, "5": {"exist": true, "char_ranges": [[113, 336]], "word_ranges": [[20, 56]], "text": "En este caso hay datos importantes como el antecedente de resecci\u00f3n intestinal, la hipocalcemia y ha hipofosforemoia; esto nos debe hacer pensar en un d\u00e9ficit de vitamina D por malabsorci\u00f3n y por tanto, en una osteomalacia."}}} {"id": 107, "year": 2012, "question_id_specific": 206, "full_question": "Una ni\u00f1a de 8 a\u00f1os (caso \u00edndice) est\u00e1 diagnosticada cl\u00ednicamente como afecta de neurofibromatosis tipo 1 (NF1) o enfermedad de Von Recklinghausen con m\u00faltiples neurofibromas, manchas caf\u00e9 con leche y n\u00f3dulos de Lisch. Su padre (no diagnosticado de NF1) falleci\u00f3 por accidente de circulaci\u00f3n a los 38 a\u00f1os. La madre presenta a la exploraci\u00f3n dos manchas caf\u00e9 con leche y acude a la consulta de consejo gen\u00e9tico con su nueva pareja donde se plantea un diagn\u00f3stico gen\u00e9tico pre implantaci\u00f3n (DGP). \u00bfEst\u00e1 indicado en este caso un DGP?", "full_answer": "Lo primero es detectar la mutaci\u00f3n y una vez detectada, si la madre es portadora, proseguir con el DGP.", "type": "GEN\u00c9TICA E INMUNOLOG\u00cdA", "options": {"1": "S\u00ed, al tener la madre 2 manchas caf\u00e9 con leche es portadora y el DGP est\u00e1 indicado con estos datos.", "2": "Est\u00e1 indicado tras detectar la mutaci\u00f3n causante en el caso \u00edndice y eventualmente en su madre.", "3": "No est\u00e1 indicado pues la NF1 responde a mutaciones en el gen de la neurofibromin (17q11.2), con herencia recesiva.", "4": "No, dos manchas caf\u00e9 con leche no son diagn\u00f3sticas y su nueva pareja es muy improbable que sea portador (la NF1 es una enfermedad poco frecuente).", "5": "Est\u00e1 indicado con estos datos un DGP consistente en seleccionar embriones in vitro, para implantar en el \u00fatero materno aquellos sin la mutaci\u00f3n."}, "correct_option": 2, "explanations": {"1": {"exist": false, "char_ranges": [], "word_ranges": [], "text": ""}, "2": {"exist": true, "char_ranges": [[0, 103]], "word_ranges": [[0, 19]], "text": "Lo primero es detectar la mutaci\u00f3n y una vez detectada, si la madre es portadora, proseguir con el DGP."}, "3": {"exist": false, "char_ranges": [], "word_ranges": [], "text": ""}, "4": {"exist": false, "char_ranges": [], "word_ranges": [], "text": ""}, "5": {"exist": false, "char_ranges": [], "word_ranges": [], "text": ""}}} {"id": 114, "year": 2012, "question_id_specific": 96, "full_question": "Una mujer de 43 a\u00f1os consulta a su m\u00e9dico de Atenci\u00f3n Primaria por cansancio, gingivorragia y petequias. Se realiza anal\u00edtica en la que destaca: anemia de 8 gr/dl, trombopenia de 4000/microlitro y leucopenia de 1200/microlitro con neutropenia absoluta. En estudio de coagulaci\u00f3n se observa alargamiento del APTT (43\u2033), actividad de la protrombina disminuida (55%), hipofibrinogenemia (98 mg/dl) y presencia de concentraci\u00f3n elevada de D-Dimero y mon\u00f3meros de fibrina. Se remite para estudio hematol\u00f3gico urgente, realiz\u00e1ndose un aspirado de m\u00e9dula \u00f3sea en el que se observa infiltraci\u00f3n masiva por elementos inmaduros de n\u00facleo hendido y bastones de Auer en el citoplasma. \u00bfCu\u00e1l es el diagn\u00f3stico m\u00e1s probable en este paciente?", "full_answer": "Tras leer el enunciado tan largo, quedarse sin aire, repasarlo y agobiarnos, llega el momento de echarle cara y agarrarnos a algo que sea muy caracter\u00edstico. Una mujer con pancitopenia. Una m\u00e9dula \u00f3sea que muestra infiltraci\u00f3n masiva por elementos inmaduros que est\u00e1n proliferando sin parar\u2026\u00bfcon bastones de Auer? Leucemia aguda promieloc\u00edtica. Esos bastones son la fusi\u00f3n de gr\u00e1nulos primarios de los mielocitos en desarrollo. Lo dicen Robin&Cotran en la p\u00e1gina 696 de su libro \u00abPatolog\u00eda general\u00bb [1]. Tambi\u00e9n lo dicen Carr y Rodak en su libro \u00abAtlas de hematolog\u00eda cl\u00ednica\u00bb en la p\u00e1gina 157 [2].", "type": "HEMATOLOG\u00cdA", "options": {"1": "Leucemia mieloide cr\u00f3nica.", "2": "Leucemia aguda promieloc\u00edtica.", "3": "Leucemia aguda monobl\u00e1stica.", "4": "Leucemia linfoide cr\u00f3nica.", "5": "Leucemia linfoide aguda."}, "correct_option": 2, "explanations": {"1": {"exist": false, "char_ranges": [], "word_ranges": [], "text": ""}, "2": {"exist": true, "char_ranges": [[158, 427]], "word_ranges": [[26, 64]], "text": "Una mujer con pancitopenia. Una m\u00e9dula \u00f3sea que muestra infiltraci\u00f3n masiva por elementos inmaduros que est\u00e1n proliferando sin parar\u2026\u00bfcon bastones de Auer? Leucemia aguda promieloc\u00edtica. Esos bastones son la fusi\u00f3n de gr\u00e1nulos primarios de los mielocitos en desarrollo."}, "3": {"exist": false, "char_ranges": [], "word_ranges": [], "text": ""}, "4": {"exist": false, "char_ranges": [], "word_ranges": [], "text": ""}, "5": {"exist": false, "char_ranges": [], "word_ranges": [], "text": ""}}} {"id": 61, "year": 2011, "question_id_specific": 92, "full_question": "Un hombre de 30 a\u00f1os sufre ca\u00edda de una altura de 2 metros de pie. A su llegada a Urgencias, est\u00e1 cyo, con sondaje vesical con orina clara. Muestra tumefacci\u00f3n de tal\u00f3n izquierdo, as\u00ed como hormigueos leves por cara anterior de muslo derecho y escroto, as\u00ed como disestesias en ambos pies. El paciente se despide del camillero con un apret\u00f3n de manos, con una palmada simult\u00e1nea en el hombro. La rotaci\u00f3n de los MMII en la camilla cogidos por los tobillos es indolora y la compresi\u00f3n de la pelvis es asintom\u00e1tica. A continuaci\u00f3n:", "full_answer": "Por los s\u00edntomas que cuentan, hormigueo en cara anterior de muslo, escroto y disestesias en pies, parece existir una afectaci\u00f3n vertebral a nivel L2-3 y ra\u00edces sacras, con lo que lo correcto ser\u00eda realizar un examen radiol\u00f3gico sin estar movilizando al paciente. Pero s\u00ed hay que movilizarlo para explorarlo, evidentemente, en bloque. La columna cervical parece estar indemne porque el paciente moviliza adecuadamente los dos brazos, sin parestesias.", "type": "ANESTESIOLOG\u00cdA Y CUIDADOS CR\u00cdTICOS", "options": {"1": "Lo sentaremos en la camilla para poder explorar la columna cervical.", "2": "Remitiremos a Rx para realizar estudio de extremidades inferiores.", "3": "Palparemos el abdomen y lo rotaremos en bloque para palpar la columna toraco-lumbar.", "4": "Le colocaremos un collar\u00edn cervical y remitiremos a radiolog\u00eda para estudio de imagen de columna cervical y extremidades inferiores.", "5": "Solicitaremos una eco abdominal."}, "correct_option": 3, "explanations": {"1": {"exist": false, "char_ranges": [], "word_ranges": [], "text": ""}, "2": {"exist": false, "char_ranges": [], "word_ranges": [], "text": ""}, "3": {"exist": true, "char_ranges": [[268, 449]], "word_ranges": [[43, 68]], "text": "s\u00ed hay que movilizarlo para explorarlo, evidentemente, en bloque. La columna cervical parece estar indemne porque el paciente moviliza adecuadamente los dos brazos, sin parestesias."}, "4": {"exist": false, "char_ranges": [], "word_ranges": [], "text": ""}, "5": {"exist": false, "char_ranges": [], "word_ranges": [], "text": ""}}} {"id": 231, "year": 2014, "question_id_specific": 180, "full_question": "Lactante de 1 mes de vida que acude a urgencias del hospital remitido por su pediatra por ictericia. Refiere \u00e9sta desde hace 10 d\u00edas y ha ido en aumento. En la anal\u00edtica de sangre destaca una bilirrubina total de 7mg/dl siendo la bilirrubina indirecta de 1,5mg/dl. La causa m\u00e1s probable, de entre las siguientes, de esta ictericia es:", "full_answer": "El aumento de la bilirrubina directa nos descarta de inmediato el resto de las opciones donde el aumento de bilirrubina total ser\u00eda a expensas de la bilirrubina indirecta. La atresia de v\u00edas biliares es la principal indicaci\u00f3n de trasplante hep\u00e1tico infantil.", "type": "PEDIATR\u00cdA", "options": {"1": "Atresia de v\u00edas biliares.", "2": "Ictericia por lactancia materna.", "3": "Isoinmunizaci\u00f3n 0-A de aparici\u00f3n tard\u00eda.", "4": "Enfermedad de Gilbert.", "5": "Esferocitosis hereditaria."}, "correct_option": 1, "explanations": {"1": {"exist": true, "char_ranges": [[172, 259]], "word_ranges": [[28, 41]], "text": "La atresia de v\u00edas biliares es la principal indicaci\u00f3n de trasplante hep\u00e1tico infantil."}, "2": {"exist": true, "char_ranges": [[0, 171]], "word_ranges": [[0, 28]], "text": "El aumento de la bilirrubina directa nos descarta de inmediato el resto de las opciones donde el aumento de bilirrubina total ser\u00eda a expensas de la bilirrubina indirecta."}, "3": {"exist": true, "char_ranges": [[0, 171]], "word_ranges": [[0, 28]], "text": "El aumento de la bilirrubina directa nos descarta de inmediato el resto de las opciones donde el aumento de bilirrubina total ser\u00eda a expensas de la bilirrubina indirecta."}, "4": {"exist": true, "char_ranges": [[0, 171]], "word_ranges": [[0, 28]], "text": "El aumento de la bilirrubina directa nos descarta de inmediato el resto de las opciones donde el aumento de bilirrubina total ser\u00eda a expensas de la bilirrubina indirecta."}, "5": {"exist": true, "char_ranges": [[0, 171]], "word_ranges": [[0, 28]], "text": "El aumento de la bilirrubina directa nos descarta de inmediato el resto de las opciones donde el aumento de bilirrubina total ser\u00eda a expensas de la bilirrubina indirecta."}}} {"id": 87, "year": 2012, "question_id_specific": 54, "full_question": "Un paciente de 81 a\u00f1os consulta por un episodio de s\u00edncope. Refiere disnea de esfuerzo desde hace un a\u00f1o. En la auscultaci\u00f3n cardiaca destaca un soplo sist\u00f3lico 3/6 en borde esternal izquierdo que irradia a car\u00f3tidas y punta. \u00bfQu\u00e9 patolog\u00eda le parece m\u00e1s probable?", "full_answer": "Pregunta muy f\u00e1cil. Nos est\u00e1n describiendo el \u201cretrato\u201d de una estenosis a\u00f3rtica degenerativa\u2026 Paciente anciano con cl\u00ednica de disnea y s\u00edncope, y soplo sist\u00f3lico irradiado a car\u00f3tidas y punta, todo ello t\u00edpico de esta enfermedad.", "type": "CARDIOLOG\u00cdA Y CIRUG\u00cdA VASCULAR", "options": {"1": "Bloqueo aur\u00edculo-ventricular de tercer grado.", "2": "Estenosis valvular a\u00f3rtica degenerativa.", "3": "Insuficiencia valvular mitral.", "4": "Miocardiopat\u00eda hipertr\u00f3fica.", "5": "Miocardiopat\u00eda dilatada."}, "correct_option": 2, "explanations": {"1": {"exist": false, "char_ranges": [], "word_ranges": [], "text": ""}, "2": {"exist": true, "char_ranges": [[95, 230]], "word_ranges": [[13, 35]], "text": "Paciente anciano con cl\u00ednica de disnea y s\u00edncope, y soplo sist\u00f3lico irradiado a car\u00f3tidas y punta, todo ello t\u00edpico de esta enfermedad."}, "3": {"exist": false, "char_ranges": [], "word_ranges": [], "text": ""}, "4": {"exist": false, "char_ranges": [], "word_ranges": [], "text": ""}, "5": {"exist": false, "char_ranges": [], "word_ranges": [], "text": ""}}} {"id": 102, "year": 2012, "question_id_specific": 87, "full_question": "Mujer de 76 a\u00f1os con obesidad y diabetes mellitus tipo 2 en tratamiento con metformina. Acude a urgencias porque desde hace 3 d\u00edas presenta fiebre de 38\u00baC, polaquiuria, disuria, intensa sed y disminuci\u00f3n progresiva de su nivel de conciencia. En la exploraci\u00f3n f\u00edsica se aprecian signos de deshidrataci\u00f3n, presi\u00f3n arterial 95/54 mmHg y no hay signos de focalidad neurol\u00f3gica. En la anal\u00edtica destaca leucocitosis, creatinina de 1.8 mg/dL \u00bf Cu\u00e1l es el diagn\u00f3stico m\u00e1s probable?", "full_answer": "Pregunta f\u00e1cil de complicaciones agudas de la diabetes mellitus.", "type": "ENDOCRINOLOG\u00cdA", "options": {"1": "Coma hipergluc\u00e9mico hiperosmolar no cet\u00f3sico.", "2": "Insuficiencia card\u00edaca.", "3": "Hematoma subdural.", "4": "Insuficiencia renal de causa obstructiva.", "5": "Cetoacidosis diab\u00e9tica con coma."}, "correct_option": 1, "explanations": {"1": {"exist": false, "char_ranges": [], "word_ranges": [], "text": ""}, "2": {"exist": false, "char_ranges": [], "word_ranges": [], "text": ""}, "3": {"exist": false, "char_ranges": [], "word_ranges": [], "text": ""}, "4": {"exist": false, "char_ranges": [], "word_ranges": [], "text": ""}, "5": {"exist": false, "char_ranges": [], "word_ranges": [], "text": ""}}} {"id": 512, "year": 2021, "question_id_specific": 153, "full_question": "Paciente diagnosticado de diverticulitis aguda, tratado con amoxicilina/clavul\u00e1nico. Al cabo de 5 d\u00edas comienza con fiebre y diarrea. Se le diagnostica de colitis por Clostridium difficile, se suspende la amoxicilina/clavul\u00e1nico y se pauta metronidazol. A los 4 d\u00edas no ha respondido al metronidazol, pero se encuentra cl\u00ednicamente estable. El siguiente paso del tratamiento es:", "full_answer": "El tratamiento de elecci\u00f3n de la infecci\u00f3n por Clostridioides difficile es la vancomicina v\u00eda oral 125mg/6h durante 10 d\u00edas. El metronidazol ha quedado relegado a situaciones en las que no se dispone de vancomicina.", "type": "DIGESTIVO", "options": {"1": "Cambiar el metronidazol por vancomicina oral (125 mg, cuatro veces al d\u00eda durante 10 d\u00edas).", "2": "Cambiar el metronidazol por piperacilina/ tazobactam (4 g de piperacilina/0,5 g de tazobactam cada 8 horas).", "3": "Indicar una colectom\u00eda subtotal.", "4": "Realizar una ileostom\u00eda en asa con lavado anter\u00f3grado de polietilenglicol.", "5": NaN}, "correct_option": 1, "explanations": {"1": {"exist": true, "char_ranges": [[0, 124]], "word_ranges": [[0, 19]], "text": "El tratamiento de elecci\u00f3n de la infecci\u00f3n por Clostridioides difficile es la vancomicina v\u00eda oral 125mg/6h durante 10 d\u00edas."}, "2": {"exist": false, "char_ranges": [], "word_ranges": [], "text": ""}, "3": {"exist": false, "char_ranges": [], "word_ranges": [], "text": ""}, "4": {"exist": false, "char_ranges": [], "word_ranges": [], "text": ""}, "5": {"exist": false, "char_ranges": [], "word_ranges": [], "text": ""}}} {"id": 218, "year": 2014, "question_id_specific": 123, "full_question": "Hombre de 30 a\u00f1os sin antecedentes de inter\u00e9s. Acude a consulta por la presencia de unas lesiones eritemato-viol\u00e1ceas de peque\u00f1o tama\u00f1o que a la palpaci\u00f3n parecen sobreelevadas, en regi\u00f3n pretibial. El estudio anal\u00edtico muestra un hemograma y estudio de coagulaci\u00f3n sin alteraciones, y en la bioqu\u00edmica, la creatinina y los iones se encuentran tambi\u00e9n dentro del rango de normalidad. El estudio del sedimento urinario demuestra hematuria, por la que el paciente ya hab\u00eda sido estudiado en otras ocasiones, sin obtener un diagn\u00f3stico definitivo. Respecto a la entidad que usted sospecha en este caso, es FALSO que:", "full_answer": "Nos est\u00e1n hablando con alta probabilidad de una glomerulonefritis mesangial IgA o enfermedad de Berger. Por tanto, vamos a ir descartando opciones una por una: 1: Verdadera. Se encuentra elevaci\u00f3n de IgA s\u00e9rica hasta en un 50% de los casos. 2: Verdadera. Los dep\u00f3sitos mesangiales de IgA se presentan en casi un 100% de los casos. 3: Esta opci\u00f3n es falsa, ya que esta glomerulonefritis se manifiesta cl\u00e1sicamente con s\u00edndrome nefr\u00edtico y no nefr\u00f3tico (aunque en algunos casos poco frecuentes s\u00ed que aparece proteinuria en rango nefr\u00f3tico, pero en el MIR no preguntan por esos casos raros!). 4: Al principio esta opci\u00f3n me generaba dudas, pero buscando en la literatura, es cierto que la evoluci\u00f3n a insuficiencia renal (seg\u00fan \u00faltimas series) se produce en torno a un 25% de los casos, por lo que esta opci\u00f3n es verdadera. 5: La biopsia cut\u00e1nea, por su mayor facilidad de realizaci\u00f3n ante la biopsia renal, es la t\u00e9cnica diagn\u00f3stica de elecci\u00f3n (se biopsian las lesiones de la piel que constituyen la p\u00farpura de Schonlein-Henoch, tan frecuentemente asociada a esta entidad y que presenta el paciente del caso).", "type": "NEFROLOG\u00cdA", "options": {"1": "En el 20 al 50% de los casos existe elevaci\u00f3n de la concentraci\u00f3n s\u00e9rica de IgA.", "2": "En la biopsia renal son caracter\u00edsticos los dep\u00f3sitos mesangiales de IgA.", "3": "Es frecuente la existencia de proteinuria en rango nefr\u00f3tico.", "4": "Se considera una entidad benigna ya que menos de 1/3 de los pacientes evolucionan a insuficiencia renal.", "5": "La biopsia cut\u00e1nea permite establecer el diagn\u00f3stico hasta en la mitad de los casos."}, "correct_option": 3, "explanations": {"1": {"exist": true, "char_ranges": [[160, 240]], "word_ranges": [[25, 40]], "text": "1: Verdadera. Se encuentra elevaci\u00f3n de IgA s\u00e9rica hasta en un 50% de los casos."}, "2": {"exist": true, "char_ranges": [[241, 330]], "word_ranges": [[40, 56]], "text": "2: Verdadera. Los dep\u00f3sitos mesangiales de IgA se presentan en casi un 100% de los casos."}, "3": {"exist": true, "char_ranges": [[331, 450]], "word_ranges": [[56, 74]], "text": "3: Esta opci\u00f3n es falsa, ya que esta glomerulonefritis se manifiesta cl\u00e1sicamente con s\u00edndrome nefr\u00edtico y no nefr\u00f3tico"}, "4": {"exist": true, "char_ranges": [[591, 821]], "word_ranges": [[97, 138]], "text": "4: Al principio esta opci\u00f3n me generaba dudas, pero buscando en la literatura, es cierto que la evoluci\u00f3n a insuficiencia renal (seg\u00fan \u00faltimas series) se produce en torno a un 25% de los casos, por lo que esta opci\u00f3n es verdadera."}, "5": {"exist": true, "char_ranges": [[822, 1109]], "word_ranges": [[138, 184]], "text": "5: La biopsia cut\u00e1nea, por su mayor facilidad de realizaci\u00f3n ante la biopsia renal, es la t\u00e9cnica diagn\u00f3stica de elecci\u00f3n (se biopsian las lesiones de la piel que constituyen la p\u00farpura de Schonlein-Henoch, tan frecuentemente asociada a esta entidad y que presenta el paciente del caso)."}}} {"id": 493, "year": 2020, "question_id_specific": 107, "full_question": "Paciente de 18 a\u00f1os que acude a urgencias con epistaxis de varios d\u00edas de evoluci\u00f3n, sin antecedentes personales ni familiares de inter\u00e9s. En la exploraci\u00f3n est\u00e1 afebril, se observan equimosis m\u00faltiples, no se palpa esplenomegalia. Anal\u00edtica: Leucocitos 7,2 x103/\u03bcL, Hb 12,3 g/dL, Plaquetas 6,0 x103/\u03bcL. La trombocitopenia se confirma en el frotis, donde se observan plaquetas de tama\u00f1o aumentado. Estudio de coagulaci\u00f3n y bioqu\u00edmica normales. \u00bfCu\u00e1l considera el diagn\u00f3stico m\u00e1s probable?:", "full_answer": "Si quisieran que respondi\u00e9ramos que es una PTT (respuesta 1) probablemente asociar\u00edan anemia y de seguro que en el frotis nos hablar\u00edan de esquistocitos. Por no decir, que probablemente la cl\u00ednica del paciente ser\u00eda mucho m\u00e1s que una epistaxis. Las otras dos respuestas, no ten\u00e9is que dudar con ellas. Nada orienta a eso.", "type": "HEMATOLOGIA", "options": {"1": "P\u00farpura tromb\u00f3tica trombocitop\u00e9nica.", "2": "Coagulaci\u00f3n intravascular diseminada.", "3": "Trombocitopenia inducida por infecci\u00f3n.", "4": "Trombocitopenia inmune primaria.", "5": NaN}, "correct_option": 4, "explanations": {"1": {"exist": true, "char_ranges": [[0, 153]], "word_ranges": [[0, 24]], "text": "Si quisieran que respondi\u00e9ramos que es una PTT (respuesta 1) probablemente asociar\u00edan anemia y de seguro que en el frotis nos hablar\u00edan de esquistocitos."}, "2": {"exist": true, "char_ranges": [[245, 321]], "word_ranges": [[39, 53]], "text": "Las otras dos respuestas, no ten\u00e9is que dudar con ellas. Nada orienta a eso."}, "3": {"exist": true, "char_ranges": [[245, 321]], "word_ranges": [[39, 53]], "text": "Las otras dos respuestas, no ten\u00e9is que dudar con ellas. Nada orienta a eso."}, "4": {"exist": false, "char_ranges": [], "word_ranges": [], "text": ""}, "5": {"exist": false, "char_ranges": [], "word_ranges": [], "text": ""}}} {"id": 566, "year": 2022, "question_id_specific": 149, "full_question": "Mujer de 67 a\u00f1os con enfermedad renal cr\u00f3nica secundaria a nefropat\u00eda diab\u00e9tica. En la actualidad presenta una creatinina de 3,2 mg/dl con filtrado glomerular estimado (CKD-EPI) 14 ml/min. \u00bfCu\u00e1l de las siguientes afirmaciones es cierta?:", "full_answer": "Un filtrado glomerular inferior a 15 ml/min se clasifica como estadio 5 (G5), indicando que la enfermedad renal se encuentra en estadio terminal y debe plantearse el inicio de terapia sustitutiva (opci\u00f3n 2 cierta, opci\u00f3n 1 falsa). En esta situaci\u00f3n no es esperable que un buen control gluc\u00e9mico pueda revertir el da\u00f1o renal, aunque s\u00ed podr\u00eda ayudar a retrasar la progresi\u00f3n (opci\u00f3n 3 falsa). Estos pacientes suelen presentar una albuminuria estadio A3 (superior a 300 mg/24 h), pudiendo llegar al rango nefr\u00f3tico (opci\u00f3n 4 falsa).", "type": "NEFROLOG\u00cdA", "options": {"1": "Presenta una enfermedad renal cr\u00f3nica estadio 3 y debe iniciar quelantes del f\u00f3sforo.", "2": "Presenta una enfermedad renal cr\u00f3nica estadio 5 y debe considerarse el tratamiento renal sustitutivo.", "3": "Un buen control gluc\u00e9mico puede revertir la insuficiencia renal.", "4": "Es poco probable que presente una proteinuria superior a 500 mg/24 h.", "5": NaN}, "correct_option": 2, "explanations": {"1": {"exist": true, "char_ranges": [[0, 230]], "word_ranges": [[0, 37]], "text": "Un filtrado glomerular inferior a 15 ml/min se clasifica como estadio 5 (G5), indicando que la enfermedad renal se encuentra en estadio terminal y debe plantearse el inicio de terapia sustitutiva (opci\u00f3n 2 cierta, opci\u00f3n 1 falsa)."}, "2": {"exist": true, "char_ranges": [[0, 230]], "word_ranges": [[0, 37]], "text": "Un filtrado glomerular inferior a 15 ml/min se clasifica como estadio 5 (G5), indicando que la enfermedad renal se encuentra en estadio terminal y debe plantearse el inicio de terapia sustitutiva (opci\u00f3n 2 cierta, opci\u00f3n 1 falsa)."}, "3": {"exist": true, "char_ranges": [[231, 391]], "word_ranges": [[37, 64]], "text": "En esta situaci\u00f3n no es esperable que un buen control gluc\u00e9mico pueda revertir el da\u00f1o renal, aunque s\u00ed podr\u00eda ayudar a retrasar la progresi\u00f3n (opci\u00f3n 3 falsa)."}, "4": {"exist": true, "char_ranges": [[392, 530]], "word_ranges": [[64, 85]], "text": "Estos pacientes suelen presentar una albuminuria estadio A3 (superior a 300 mg/24 h), pudiendo llegar al rango nefr\u00f3tico (opci\u00f3n 4 falsa)."}, "5": {"exist": false, "char_ranges": [], "word_ranges": [], "text": ""}}} {"id": 254, "year": 2014, "question_id_specific": 89, "full_question": "Un chico de 17 a\u00f1os, deportista y sin antecedentes de inter\u00e9s, acude al m\u00e9dico por presentar ictericia conjuntival ocasionalmente. Niega ingesta de f\u00e1rmacos y dolor abdominal. En los an\u00e1lisis realizados se observan cifras de ALT, AST, GGT y FA normales, con bilirrubina total de 3,2 mg/dl y bilirrubina directa de 0,4 mg/dl. No tiene anemia y el h\u00edgado es ecogr\u00e1ficamente normal. \u00bfCu\u00e1l es su diagn\u00f3stico?", "full_answer": "Un aumento de la bilirrubina indirecta menor de 5 (de manera intermitente, bajo situaciones de estr\u00e9s) en un chico joven sano nos debe hacer pensar en un s\u00edndrome de Gilbert. En el s\u00edndrome de Rotor, Dubin-Johnson y en la coledocolitiasis hay aumento de la bilirrubina directa. De tratarse de una hepatitis aguda deber\u00eda haber una elevaci\u00f3n de las transaminasas.", "type": "APARATO DIGESTIVO", "options": {"1": "S\u00edndrome de Rotor.", "2": "Coledocolitiasis.", "3": "S\u00edndrome de Dubin-Johnson.", "4": "Hepatitis aguda.", "5": "S\u00edndrome de Gilbert."}, "correct_option": 5, "explanations": {"1": {"exist": true, "char_ranges": [[175, 277]], "word_ranges": [[30, 46]], "text": "En el s\u00edndrome de Rotor, Dubin-Johnson y en la coledocolitiasis hay aumento de la bilirrubina directa."}, "2": {"exist": true, "char_ranges": [[175, 277]], "word_ranges": [[30, 46]], "text": "En el s\u00edndrome de Rotor, Dubin-Johnson y en la coledocolitiasis hay aumento de la bilirrubina directa."}, "3": {"exist": true, "char_ranges": [[175, 277]], "word_ranges": [[30, 46]], "text": "En el s\u00edndrome de Rotor, Dubin-Johnson y en la coledocolitiasis hay aumento de la bilirrubina directa."}, "4": {"exist": true, "char_ranges": [[278, 362]], "word_ranges": [[46, 59]], "text": "De tratarse de una hepatitis aguda deber\u00eda haber una elevaci\u00f3n de las transaminasas."}, "5": {"exist": false, "char_ranges": [], "word_ranges": [], "text": ""}}} {"id": 597, "year": 2022, "question_id_specific": 180, "full_question": "Mujer de 26 a\u00f1os diagnosticada de lupus eritematoso sist\u00e9mico, en tratamiento con hidroxicloroquina, que consulta por sensaci\u00f3n de debilidad generalizada que se ha instaurado progresivamente en los \u00faltimos 15 d\u00edas. En la exploraci\u00f3n f\u00edsica se aprecia palidez cut\u00e1nea y en la anal\u00edtica destaca Hb 7,4 gr/dL, Hcto 31 %, VCM 108. \u00bfCu\u00e1l de las siguientes determinaciones ser\u00e1 m\u00e1s \u00fatil para decidir la actuaci\u00f3n?", "full_answer": "En esta pregunta, nos plantean una paciente con lupus eritematoso sist\u00e9mico que presenta una anemia hemol\u00edtica autoinmune. Para ello, la mejor opci\u00f3n ser\u00eda solicitar unos reticulocitos, no obstante, como no se encuentra entre las opciones, deber\u00edamos solicitar un test de Coombs, por lo que la respuesta correcta es la 2. Si bien, nos quieren confundir con un VCM 108, para que pensemos en una anemia megalob\u00e1stica, el VCM no es excesivamente elevado, la instauraci\u00f3n de la anemia en nuestra paciente ha sido r\u00e1pida y no cuadrar\u00eda con el LES ni la forma de presentaci\u00f3n. Los anticuerpos antinucleares se utilizan para el diagn\u00f3stico, pero no para el seguimiento del LES.", "type": "REUMATOLOG\u00cdA", "options": {"1": "Haptoglobina.", "2": "Test de Coombs.", "3": "Vitamina B12.", "4": "Anticuerpos antinucleares.", "5": NaN}, "correct_option": 2, "explanations": {"1": {"exist": false, "char_ranges": [], "word_ranges": [], "text": ""}, "2": {"exist": true, "char_ranges": [[0, 322]], "word_ranges": [[0, 50]], "text": "En esta pregunta, nos plantean una paciente con lupus eritematoso sist\u00e9mico que presenta una anemia hemol\u00edtica autoinmune. Para ello, la mejor opci\u00f3n ser\u00eda solicitar unos reticulocitos, no obstante, como no se encuentra entre las opciones, deber\u00edamos solicitar un test de Coombs, por lo que la respuesta correcta es la 2."}, "3": {"exist": true, "char_ranges": [[323, 571]], "word_ranges": [[50, 94]], "text": "Si bien, nos quieren confundir con un VCM 108, para que pensemos en una anemia megalob\u00e1stica, el VCM no es excesivamente elevado, la instauraci\u00f3n de la anemia en nuestra paciente ha sido r\u00e1pida y no cuadrar\u00eda con el LES ni la forma de presentaci\u00f3n."}, "4": {"exist": true, "char_ranges": [[572, 671]], "word_ranges": [[94, 109]], "text": "Los anticuerpos antinucleares se utilizan para el diagn\u00f3stico, pero no para el seguimiento del LES."}, "5": {"exist": false, "char_ranges": [], "word_ranges": [], "text": ""}}} {"id": 571, "year": 2022, "question_id_specific": 170, "full_question": "Mujer de 75 a\u00f1os en tratamiento con sertralina que acude a urgencias por un cuadro confusional. No se evidencian edemas y la tensi\u00f3n arterial es de 130/70. En la anal\u00edtica destaca Na 126 mEq/l y K 4 mEq/l, la natriuria es de 45 mEq/l y se ha descartado la ingesta de diur\u00e9ticos. \u00bfDe las siguientes cu\u00e1l es la actitud m\u00e1s correcta?:", "full_answer": "Nos presentan a una paciente con una hiponatremia moderada euvol\u00e9mica, orientando hacia un s\u00edndrome de antidiuresis inadecuada (SIAD) secundario a la sertralina, aunque ser\u00eda necesario disponer de la osmolalidad urinaria para confirmarlo. La insuficiencia suprarrenal primaria produce hiponatremia hipovol\u00e9mica (opci\u00f3n 1 incorrecta). La diabetes ins\u00edpida es la contraposici\u00f3n a esta patolog\u00eda, present\u00e1ndose habitualmente con hipernatremia (opci\u00f3n 4 incorrecta). La duda queda entre si hay que tratar de forma agresiva con suero salino hipert\u00f3nico, o ser m\u00e1s conservador indicando restricci\u00f3n h\u00eddrica. Con natremias superiores a 125 mmol/l no est\u00e1 indicado tratar con SSH salvo que el paciente tenga cl\u00ednica neurol\u00f3gica grave, que no parece el caso (describen un cuadro confusional que ser\u00eda una encefalopat\u00eda hiponatr\u00e9mica moderada), y tampoco est\u00e1 indicado restablecer cuanto antes la natremia (por el riesgo de s\u00edndrome de desmielinizaci\u00f3n osm\u00f3tica). Por tanto, la respuesta correcta ser\u00eda la 3. Lo que no me gusta de esa respuesta es que la segunda parte (\u201csi no se eleva la natremia pasar a infusi\u00f3n lenta de suero salino\u201d) es incorrecta, ya que en ese caso habr\u00eda que plantearse el uso de furosemida, tolvaptan o urea, no suero salino (y tampoco indica la tonicidad, no es lo mismo un isot\u00f3nico que un hipert\u00f3nico). En cualquier caso, no la veo muy susceptible de anularse.", "type": "NEFROLOG\u00cdA", "options": {"1": "Se trata de una enfermedad de Addison y deben administrarse corticoides de inmediato.", "2": "Administrar suero salino hipert\u00f3nico a fin de restablecer cuanto antes la natremia.", "3": "Indicar restricci\u00f3n h\u00eddrica y si no se eleva la natremia pasar a infusi\u00f3n lenta de suero salino.", "4": "Solicitar resonancia magn\u00e9tica cerebral, ya que seguramente se trata de una diabetes ins\u00edpida.", "5": NaN}, "correct_option": 3, "explanations": {"1": {"exist": true, "char_ranges": [[239, 333]], "word_ranges": [[32, 42]], "text": "La insuficiencia suprarrenal primaria produce hiponatremia hipovol\u00e9mica (opci\u00f3n 1 incorrecta)."}, "2": {"exist": true, "char_ranges": [[602, 953]], "word_ranges": [[80, 132]], "text": "Con natremias superiores a 125 mmol/l no est\u00e1 indicado tratar con SSH salvo que el paciente tenga cl\u00ednica neurol\u00f3gica grave, que no parece el caso (describen un cuadro confusional que ser\u00eda una encefalopat\u00eda hiponatr\u00e9mica moderada), y tampoco est\u00e1 indicado restablecer cuanto antes la natremia (por el riesgo de s\u00edndrome de desmielinizaci\u00f3n osm\u00f3tica)."}, "3": {"exist": false, "char_ranges": [], "word_ranges": [], "text": ""}, "4": {"exist": true, "char_ranges": [[334, 462]], "word_ranges": [[42, 58]], "text": "La diabetes ins\u00edpida es la contraposici\u00f3n a esta patolog\u00eda, present\u00e1ndose habitualmente con hipernatremia (opci\u00f3n 4 incorrecta)."}, "5": {"exist": false, "char_ranges": [], "word_ranges": [], "text": ""}}} {"id": 466, "year": 2020, "question_id_specific": 91, "full_question": "Ni\u00f1a de 11 a\u00f1os y 6 meses, premen\u00e1rquica, con escoliosis idiop\u00e1tica conocida. La radiograf\u00eda de raquis realizada hace 6 meses mostraba una curva t\u00f3rca-lumbar izquierda de 18\u00ba (\u00e1ngulo de Cobb). En la actual se observa un \u00e1ngulo de Cobb de 28\u00ba, con una maduraci\u00f3n de cresta iliaca (signo de Risser) de 2. En este momento evolutivo, \u00bfqu\u00e9 tratamiento es el m\u00e1s adecuado para intentar detener la progresi\u00f3n de la curva?:", "full_answer": "En las formas juvenil y del adolescente el tratamiento depende de los grados de deformidad: \u2022 10 \u2013 20\u00b0: observaci\u00f3n seriada. \u2022 20-50\u00b0: cors\u00e9s de termopl\u00e1stico (Boston, Cheneau o Milwaukee, seg\u00fan d\u00f3nde se localice el \u00e1pex de la curva principal). \u2022 40-50\u00b0: con Risser menor de 3 estar\u00eda indicada la artrodesis quir\u00fargica; con Risser mayor o igual de 3 el potencial remanente de crecimiento de la curva es escaso y se puede valorar la observaci\u00f3n por si tendiera a progresar en la edad adulta.", "type": "CIRUG\u00cdA ORTOP\u00c9DICA Y TRAUMATOLOG\u00cdA", "options": {"1": "Ejercicios de nataci\u00f3n.", "2": "Ejercicios de reeducaci\u00f3n postural.", "3": "Cors\u00e9 ortop\u00e9dico corrector.", "4": "La intervenci\u00f3n quir\u00fargica mediante artrodesis espinal.", "5": NaN}, "correct_option": 3, "explanations": {"1": {"exist": false, "char_ranges": [], "word_ranges": [], "text": ""}, "2": {"exist": false, "char_ranges": [], "word_ranges": [], "text": ""}, "3": {"exist": true, "char_ranges": [[127, 244]], "word_ranges": [[22, 40]], "text": "20-50\u00b0: cors\u00e9s de termopl\u00e1stico (Boston, Cheneau o Milwaukee, seg\u00fan d\u00f3nde se localice el \u00e1pex de la curva principal)."}, "4": {"exist": true, "char_ranges": [[255, 319]], "word_ranges": [[42, 52]], "text": "con Risser menor de 3 estar\u00eda indicada la artrodesis quir\u00fargica;"}, "5": {"exist": false, "char_ranges": [], "word_ranges": [], "text": ""}}} {"id": 175, "year": 2013, "question_id_specific": 53, "full_question": "Hombre de 71 a\u00f1os, camarero jubilado, no fumador, que consulta por ronquidos nocturnos. El paciente tiene un \u00edndice de masa corporal de 31,5 Kg/m2 y refiere ligera somnolencia diurna postprandial (Escala de Somnolencia de Epworth de 3) sin otros s\u00edntomas, signos o antecedentes cl\u00ednicos relevantes. El \u00edndice de apnea\u00ad-hipopnea obtenido en la polisomnograf\u00eda fue de 18. Con estos datos, \u00bfcu\u00e1l es la medida a adoptar m\u00e1s adecuada en este paciente?", "full_answer": "Con un Indice de Apneas-Hipopneas de menos de 30, y en ausencia de s\u00edntomas llamativos de SAOS y en ausencia de profesiones de riesgo como podr\u00eda ser conductor profesional, etc, lo correcto es el control de peso y el resto de medidas higi\u00e9nico-diet\u00e9ticas, con seguimiento peri\u00f3dico.", "type": "NEUMOLOG\u00cdA", "options": {"1": "Medidas higi\u00e9nico-diet\u00e9ticas y control evolutivo.", "2": "Uvulopalatofaringoplastia.", "3": "CPAP (Presi\u00f3n Positiva Continua en la V\u00eda A\u00e9rea) nasal.", "4": "Ventilaci\u00f3n mec\u00e1nica no invasiva.", "5": "Oxigenoterapia nocturna."}, "correct_option": 1, "explanations": {"1": {"exist": true, "char_ranges": [[0, 282]], "word_ranges": [[0, 46]], "text": "Con un Indice de Apneas-Hipopneas de menos de 30, y en ausencia de s\u00edntomas llamativos de SAOS y en ausencia de profesiones de riesgo como podr\u00eda ser conductor profesional, etc, lo correcto es el control de peso y el resto de medidas higi\u00e9nico-diet\u00e9ticas, con seguimiento peri\u00f3dico."}, "2": {"exist": false, "char_ranges": [], "word_ranges": [], "text": ""}, "3": {"exist": false, "char_ranges": [], "word_ranges": [], "text": ""}, "4": {"exist": false, "char_ranges": [], "word_ranges": [], "text": ""}, "5": {"exist": false, "char_ranges": [], "word_ranges": [], "text": ""}}} {"id": 91, "year": 2012, "question_id_specific": 35, "full_question": "Mujer de 26 a\u00f1os que presenta incontinencia fecal despu\u00e9s de un parto prolongado e instrumental. Se practica ecograf\u00eda endoanal apreci\u00e1ndose una secci\u00f3n del esf\u00ednter anal externo de 30\u00ba de amplitud. El estudio electrofisiol\u00f3gico demuestra una inervaci\u00f3n normal. \u00bfCu\u00e1l es el tratamiento m\u00e1s indicado?", "full_answer": "La respuesta es la 3 esfinteroplastia quir\u00fargica. La 1 no es eficaz , la 2 tampoco para una lesi\u00f3n org\u00e1nica con electromiograf\u00eda normal, la 4 no es necesaria ya que la lesi\u00f3n es s\u00f3lo del esf\u00ednter externo, la 5 no est\u00e1 indicada en una lesi\u00f3n que se puede reparar con cirug\u00eda con buenos resultados.", "type": "CIRUG\u00cdA GENERAL", "options": {"1": "Tratamiento m\u00e9dico con normas higi\u00e9nicodiet\u00e9ticas.", "2": "Biofeedback esfinteriano.", "3": "Esfinteroplastia quir\u00fargica.", "4": "Reparaci\u00f3n quir\u00fargica del suelo p\u00e9lvico.", "5": "Esf\u00ednter anal artificial."}, "correct_option": 3, "explanations": {"1": {"exist": true, "char_ranges": [[50, 67]], "word_ranges": [[7, 12]], "text": "La 1 no es eficaz"}, "2": {"exist": true, "char_ranges": [[70, 135]], "word_ranges": [[13, 23]], "text": "la 2 tampoco para una lesi\u00f3n org\u00e1nica con electromiograf\u00eda normal,"}, "3": {"exist": false, "char_ranges": [], "word_ranges": [], "text": ""}, "4": {"exist": true, "char_ranges": [[137, 203]], "word_ranges": [[23, 37]], "text": "la 4 no es necesaria ya que la lesi\u00f3n es s\u00f3lo del esf\u00ednter externo,"}, "5": {"exist": true, "char_ranges": [[205, 296]], "word_ranges": [[37, 54]], "text": "la 5 no est\u00e1 indicada en una lesi\u00f3n que se puede reparar con cirug\u00eda con buenos resultados."}}} {"id": 65, "year": 2011, "question_id_specific": 218, "full_question": "Un hombre de 56 a\u00f1os, diab\u00e9tico y con sobrepeso moderado presenta una discreta hepatomegalia. El enfermo indica que no es bebedor habitual, s\u00f3lo excepcionalmente toma algo de vino. Una biopsia de h\u00edgado muestra ac\u00famulos de vacuolas claras citopl\u00e1smicas en un 50% de los hepatocitos. Focos inflamatorios m\u00faltiples con neutr\u00f3filos, degeneraci\u00f3n hialina de Mallory y fibrosis alrededor de las v\u00e9nulas hep\u00e1ticas terminales. Se\u00f1ale la respuesta correcta en este caso:", "full_answer": "Los cambios descritos en el enunciado de la pregunta corresponden a una situaci\u00f3n precirr\u00f3tica precoz. Si bien el alcohol es el t\u00f3xico que con m\u00e1s frecuencia provoca este cuadro, no es el causante en exclusiva. La diabetes mellitus provoca alteraciones hep\u00e1ticas similares: ac\u00famulo de grasa en los hepatocitos, fibrosis centroacinar, infiltrado inflamatorio por polimorfonucleares neutr\u00f3filos y degeneraci\u00f3n hialina de mallory perinuclear. Teniendo esto en consideraci\u00f3n, podemos descartar la segunda pregunta (son ac\u00famulos de grasa intracelular, no de gluc\u00f3geno) y la cuarta (si la diabetes tambi\u00e9n produce dicha situaci\u00f3n no hay por qu\u00e9 desconfiar del paciente). Bas\u00e1ndonos en el primum non nocere y el no tener especial relevancia para el tratamiento si es una enfermedad focal o generalizada, no es necesario repetir una biopsia hep\u00e1tica que no ampliar\u00eda informaci\u00f3n, descart\u00e1ndose la primera pregunta. Afortunadamente para el hipot\u00e9tico paciente, en fase inicial estos cambios son reversibles al desaparecer la noxa, por lo que nos quedamos con la respuesta n\u00ba 5.", "type": "ANATOM\u00cdA PATOL\u00d3GICA", "options": {"1": "Debe repetirse la biopsia para saber si los cambios son difusos.", "2": "El diagn\u00f3stico es una glucogenosis hep\u00e1tica asociada a la diabetes.", "3": "Los cambios histol\u00f3gicos son irreversibles y acabar\u00e1n produciendo una cirrosis.", "4": "La lesi\u00f3n est\u00e1 causada claramente por el alcohol a pesar de la negativa del enfermo.", "5": "Es importante disminuir peso y controlar adecuadamente la diabetes para evitar el posible desarrollo de una cirrosis."}, "correct_option": 5, "explanations": {"1": {"exist": true, "char_ranges": [[665, 906]], "word_ranges": [[95, 132]], "text": "Bas\u00e1ndonos en el primum non nocere y el no tener especial relevancia para el tratamiento si es una enfermedad focal o generalizada, no es necesario repetir una biopsia hep\u00e1tica que no ampliar\u00eda informaci\u00f3n, descart\u00e1ndose la primera pregunta."}, "2": {"exist": true, "char_ranges": [[211, 664]], "word_ranges": [[35, 95]], "text": "La diabetes mellitus provoca alteraciones hep\u00e1ticas similares: ac\u00famulo de grasa en los hepatocitos, fibrosis centroacinar, infiltrado inflamatorio por polimorfonucleares neutr\u00f3filos y degeneraci\u00f3n hialina de mallory perinuclear. Teniendo esto en consideraci\u00f3n, podemos descartar la segunda pregunta (son ac\u00famulos de grasa intracelular, no de gluc\u00f3geno) y la cuarta (si la diabetes tambi\u00e9n produce dicha situaci\u00f3n no hay por qu\u00e9 desconfiar del paciente)."}, "3": {"exist": false, "char_ranges": [], "word_ranges": [], "text": ""}, "4": {"exist": true, "char_ranges": [[211, 664]], "word_ranges": [[35, 95]], "text": "La diabetes mellitus provoca alteraciones hep\u00e1ticas similares: ac\u00famulo de grasa en los hepatocitos, fibrosis centroacinar, infiltrado inflamatorio por polimorfonucleares neutr\u00f3filos y degeneraci\u00f3n hialina de mallory perinuclear. Teniendo esto en consideraci\u00f3n, podemos descartar la segunda pregunta (son ac\u00famulos de grasa intracelular, no de gluc\u00f3geno) y la cuarta (si la diabetes tambi\u00e9n produce dicha situaci\u00f3n no hay por qu\u00e9 desconfiar del paciente)."}, "5": {"exist": true, "char_ranges": [[952, 1068]], "word_ranges": [[137, 158]], "text": "en fase inicial estos cambios son reversibles al desaparecer la noxa, por lo que nos quedamos con la respuesta n\u00ba 5."}}} {"id": 50, "year": 2011, "question_id_specific": 160, "full_question": "Ni\u00f1a de 5 a\u00f1os, completamente asintom\u00e1tica, y que acude a la revisi\u00f3n habitual del pediatra. Consulta porque su madre fue operada de un tumor de tiroides y una abuela falleci\u00f3 hace 10 a\u00f1os por un feocromocitoma. La actitud terap\u00e9utica prioritaria que adoptaremos ser\u00e1:", "full_answer": "La respuesta correcta es la 4. Me parece una pregunta m\u00e1s de Endocrino que de Pediatr\u00eda. Los portadores del protooncogen suelen tener tumoraciones tiroideas precoces por lo que se suele recomendar la tiroidectom\u00eda radical antes de los 5 a\u00f1os. Obviamente esto lo he tenido que buscar\u2026no es muy frecuente en nuestra pr\u00e1ctica habitual.", "type": "PEDIATR\u00cdA", "options": {"1": "Exploraci\u00f3n f\u00edsica exhaustiva con control de la tensi\u00f3n arterial para valorar si el ni\u00f1o puede presentar alguna enfermedad familiar.", "2": "An\u00e1lisis bioqu\u00edmico con calcitonina. Si \u00e9sta es normal no es preciso m\u00e1s control a esta edad.", "3": "Seguimiento anual de los niveles de calcitonina con estimulaci\u00f3n con pentagastrina y, si se eleva, indicar una tiroidectom\u00eda profil\u00e1ctica.", "4": "Estudio gen\u00e9tico de la mutaci\u00f3n del proto-oncogen RET c634, y si es positivo se realizar\u00e1 la tiroidectom\u00eda radical profil\u00e1ctica a esta edad.", "5": "Estudio gen\u00e9tico de la mutaci\u00f3n proto-oncogen RET c634, niveles de calcitonina y PAAF (punci\u00f3n aspiraci\u00f3n con aguja fina) y si son positivas se realizar\u00e1 la tiroidectom\u00eda radical."}, "correct_option": 4, "explanations": {"1": {"exist": false, "char_ranges": [], "word_ranges": [], "text": ""}, "2": {"exist": false, "char_ranges": [], "word_ranges": [], "text": ""}, "3": {"exist": false, "char_ranges": [], "word_ranges": [], "text": ""}, "4": {"exist": true, "char_ranges": [[89, 242]], "word_ranges": [[16, 39]], "text": "Los portadores del protooncogen suelen tener tumoraciones tiroideas precoces por lo que se suele recomendar la tiroidectom\u00eda radical antes de los 5 a\u00f1os."}, "5": {"exist": false, "char_ranges": [], "word_ranges": [], "text": ""}}} {"id": 326, "year": 2016, "question_id_specific": 173, "full_question": "Hombre de 35 a\u00f1os de edad sufre una quemadura el\u00e9ctrica de alto voltaje (3000 voltios) por contacto directo con cable con su mano izquierda. Al ingreso presenta contractura en flexi\u00f3n de la mano, palidez de los dedos y ausencia de pulso radial y cubital a la palpaci\u00f3n \u00bfCu\u00e1l es la medida invasiva de urgencia a realizar?", "full_answer": "La 3. Ante la sospecha de s\u00edndrome compartimental la medida invasiva de urgencia a realizar ser\u00eda medir la presi\u00f3n intracompartimental y, seg\u00fan el resultado, proceder al tratamiento.", "type": "CUIDADOS CR\u00cdTICOS, PALIATIVOS Y URGENCIAS", "options": {"1": "Bloqueo axilar con cat\u00e9ter.", "2": "Escarotom\u00eda descompresiva.", "3": "Monitorizaci\u00f3n de presi\u00f3n intracompartimental.", "4": "Escarectom\u00eda.", "5": NaN}, "correct_option": 3, "explanations": {"1": {"exist": false, "char_ranges": [], "word_ranges": [], "text": ""}, "2": {"exist": false, "char_ranges": [], "word_ranges": [], "text": ""}, "3": {"exist": true, "char_ranges": [[6, 182]], "word_ranges": [[2, 27]], "text": "Ante la sospecha de s\u00edndrome compartimental la medida invasiva de urgencia a realizar ser\u00eda medir la presi\u00f3n intracompartimental y, seg\u00fan el resultado, proceder al tratamiento."}, "4": {"exist": false, "char_ranges": [], "word_ranges": [], "text": ""}, "5": {"exist": false, "char_ranges": [], "word_ranges": [], "text": ""}}} {"id": 354, "year": 2016, "question_id_specific": 166, "full_question": "Una embarazada de 10 semanas de gestaci\u00f3n, acude urgencias hospitalarias por presentar una presi\u00f3n arterial de 160/105 mmHg. Buen estado general, s\u00f3lo leve cefalea, motivo por el que se tom\u00f3 la presi\u00f3n arterial. Tras 4 horas de reposo presenta cifras de 150/95mmHg. Se le realiza un hemograma que es normal y una proteinuria que es negativa. \u00bfQu\u00e9 tipo de hipertensi\u00f3n presenta?", "full_answer": "La hipertensi\u00f3n cr\u00f3nica se define como aquella que est\u00e1 presente antes del embarazo o que se diagnostica antes de la semana 20 de gestaci\u00f3n. En este caso, la paciente est\u00e1 embarazada de 10 semanas por lo que la respuesta correcta es la 3.", "type": "GINECOLOG\u00cdA Y OBSTETRICIA", "options": {"1": "Preeclampsia moderada.", "2": "Hipertensi\u00f3n inducida por el embarazo.", "3": "Hipertensi\u00f3n cr\u00f3nica.", "4": "Eclampsia.", "5": NaN}, "correct_option": 3, "explanations": {"1": {"exist": false, "char_ranges": [], "word_ranges": [], "text": ""}, "2": {"exist": false, "char_ranges": [], "word_ranges": [], "text": ""}, "3": {"exist": true, "char_ranges": [[0, 238]], "word_ranges": [[0, 43]], "text": "La hipertensi\u00f3n cr\u00f3nica se define como aquella que est\u00e1 presente antes del embarazo o que se diagnostica antes de la semana 20 de gestaci\u00f3n. En este caso, la paciente est\u00e1 embarazada de 10 semanas por lo que la respuesta correcta es la 3."}, "4": {"exist": false, "char_ranges": [], "word_ranges": [], "text": ""}, "5": {"exist": false, "char_ranges": [], "word_ranges": [], "text": ""}}} {"id": 531, "year": 2021, "question_id_specific": 58, "full_question": "Ante un paciente que presenta una lesi\u00f3n \u00falcerovegetante en la enc\u00eda mandibular a nivel de la regi\u00f3n de los molares de 5 meses de evoluci\u00f3n, la aparici\u00f3n de hipoestesia en la regi\u00f3n mentoniana requiere descartar:", "full_answer": "Las parestesias mentonianas (signo de Roger) se asocian a met\u00e1stasis \u00f3seas (no es paraneopl\u00e1sico sino infiltraci\u00f3n directa). En este caso lo que describen es un tumor oral creciendo localmente e infiltrando el canal mandibular, creo.", "type": "ONCOLOG\u00cdA", "options": {"1": "Una neuropat\u00eda paraneopl\u00e1sica.", "2": "El diagn\u00f3stico de sarcoidosis con par\u00e1lisis facial \u00aben b\u00e1scula\u00bb.", "3": "Infecci\u00f3n por virus herpes en paciente inmunodeprimido.", "4": "El diagn\u00f3stico de carcinoma con infiltraci\u00f3n del canal mandibular.", "5": NaN}, "correct_option": 4, "explanations": {"1": {"exist": false, "char_ranges": [], "word_ranges": [], "text": ""}, "2": {"exist": false, "char_ranges": [], "word_ranges": [], "text": ""}, "3": {"exist": false, "char_ranges": [], "word_ranges": [], "text": ""}, "4": {"exist": true, "char_ranges": [[0, 233]], "word_ranges": [[0, 35]], "text": "Las parestesias mentonianas (signo de Roger) se asocian a met\u00e1stasis \u00f3seas (no es paraneopl\u00e1sico sino infiltraci\u00f3n directa). En este caso lo que describen es un tumor oral creciendo localmente e infiltrando el canal mandibular, creo."}, "5": {"exist": false, "char_ranges": [], "word_ranges": [], "text": ""}}} {"id": 93, "year": 2012, "question_id_specific": 31, "full_question": "Un muchacho de 17 a\u00f1os de edad con enfermedad de Crohn con afectaci\u00f3n col\u00f3nica de 2 a\u00f1os de evoluci\u00f3n, en tratamiento de mantenimiento con azatioprina, consulta por la aparici\u00f3n desde hace 5 d\u00edas de n\u00f3dulos subcut\u00e1neos rojo viol\u00e1ceos, calientes, dolorosos, de localizaci\u00f3n pretibial, bilaterales, asociada a un inrcemento del n\u00ba de deposiciones y dolor abdominal. La actitud m\u00e1s adecuada en este caso es:", "full_answer": "2 y 4 se descartan solas. Nos est\u00e1n explicando un eritema nodoso, as\u00ed que s\u00ed recomendariamos reposo relativo (pero lo de los antidepresivos no tiene ning\u00fan sentido, as\u00ed que descartamos tambi\u00e9n (1)). Aunque el eritema nudoso puede ser secundario a un proceso maligno, es m\u00e1s frecuente que se deba a la misma enfermedad de Crohn (adem\u00e1s en un chico de 17 a\u00f1os ser\u00eda extremadamente raro) as\u00ed que, si el cuadro adem\u00e1s se acompa\u00f1a de un brote de enfermedad inflamatoria intestina, la \u00faltima opci\u00f3n (5) parece la m\u00e1s correcta.", "type": "DERMATOLOG\u00cdA, VENEREOLOG\u00cdA Y CIRUG\u00cdA PL\u00c1STICA", "options": {"1": "Recomendar reposo relativo y pa\u00f1os calientes en ambas piernas y a\u00f1adir tratamiento antidepresivo.", "2": "Biopsiar zonas de piel alejados de las \u00e1reas lesionadas y pautar analg\u00e9sicos opioides de entrada.", "3": "Sospechar la existencia de una lesi\u00f3n tumoral maligna intestinal como desencadenante del proceso cut\u00e1neo.", "4": "Sospechar isquemia bilateral de extremidades inferiores de origen medicamentoso.", "5": "Ajuste del tratamiento de la enfermedad intestinal."}, "correct_option": 5, "explanations": {"1": {"exist": true, "char_ranges": [[26, 198]], "word_ranges": [[6, 32]], "text": "Nos est\u00e1n explicando un eritema nodoso, as\u00ed que s\u00ed recomendariamos reposo relativo (pero lo de los antidepresivos no tiene ning\u00fan sentido, as\u00ed que descartamos tambi\u00e9n (1))."}, "2": {"exist": false, "char_ranges": [], "word_ranges": [], "text": ""}, "3": {"exist": true, "char_ranges": [[199, 384]], "word_ranges": [[32, 65]], "text": "Aunque el eritema nudoso puede ser secundario a un proceso maligno, es m\u00e1s frecuente que se deba a la misma enfermedad de Crohn (adem\u00e1s en un chico de 17 a\u00f1os ser\u00eda extremadamente raro)"}, "4": {"exist": false, "char_ranges": [], "word_ranges": [], "text": ""}, "5": {"exist": true, "char_ranges": [[394, 520]], "word_ranges": [[67, 88]], "text": "si el cuadro adem\u00e1s se acompa\u00f1a de un brote de enfermedad inflamatoria intestina, la \u00faltima opci\u00f3n (5) parece la m\u00e1s correcta."}}} {"id": 390, "year": 2016, "question_id_specific": 188, "full_question": "Un hombre de 80 a\u00f1os es llevado a Urgencias por la polic\u00eda a encontrarle por la noche, en la calle, a varios bloques de su domicilio, algo perdido. Dice al m\u00e9dico de Urgencias \u00abme encuentro bien, estaba paseando y me he perdido\u00bb. La revisi\u00f3n, por sistemas, es anodina. Al preguntarle por su medicaci\u00f3n, no recuerda. En su historial constan glipizida, atenolol y acenocumarol. Ha estado en Urgencias 3 veces en los \u00faltimos tres meses por \u00abmareo\u00bb, dolor lumbar y un INR de 5. Refiere independencia en actividades b\u00e1sicas e instrumentales de la vida diaria. No hay evidencia de familiares o amigos cercanos. Varias referencias del trabajo social del \u00e1rea indican que rechaza ayudas o visitas m\u00e9dicas o de enfermer\u00eda. En la exploraci\u00f3n f\u00edsica, \u00fanicamente destaca temperatura de 37.7, IMC de 16, PA de 160/90, FC de 90 y SO2 de 99%. Tiene p\u00e9rdida global de masa muscular. Est\u00e1 vestido con la bata de casa y huele a orina. Rechaza contestar a las preguntas del Minimental. En las pruebas de laboratorio destacan 13.000 leucos, Glucemia 300, hemoglobina glicosilada 11%, INR 0.9, en orina m\u00e1s de 50 leucocitos/campo, nitritos+ y urocultivo > 100.000 bacilos Gram negativos. Es ingresado en el hospital, recibe antibi\u00f3ticos, insulina y se reinicia acenocumarol con buena evoluci\u00f3n. Est\u00e1 estable y quiere irse a su domicilio. La enfermera de planta dice que es incapaz de autoadministrarse la insulina. \u00bfCu\u00e1l de los siguientes es el pr\u00f3ximo paso para determinar la seguridad del paciente en su domicilio?", "full_answer": "Evaluar la capacidad no tiene que ver con quitar la autonom\u00eda al paciente, sino asegurarnos de que el paciente comprende y es consciente de las consecuencias de sus decisiones. De hecho es una medida de protecci\u00f3n de los pacientes m\u00e1s fr\u00e1giles para asegurar los mejores cuidados. Esto no lo hace un psiquiatra, sino el m\u00e9dico a cargo. Hay varios tests o entrevistas semiestructuradas para valorar la capacidad de los pacientes, y todos los m\u00e9dicos deber\u00edan conocer estos instrumentos. Es importante que sea el m\u00e9dico a cargo el que realiza esta valoraci\u00f3n, porque es el que previamente informa al paciente (consentimiento informado) del plan terap\u00e9utico y es el que est\u00e1 mejor capacitado para evaluar si el paciente ha entendido todo: cu\u00e1l es el problema m\u00e9dico, qu\u00e9 tratamiento se le propone, las alternativas si las hubiera, tener opci\u00f3n de aceptar alguna o rechazarlas y por qu\u00e9, consecuencias de tomar esas decisiones (tambi\u00e9n las de no querer tratamiento), incluso valorar psicopatolog\u00eda susceptible de un PIC a Psiquiatr\u00eda.", "type": "PSIQUIATR\u00cdA", "options": {"1": "Referir a Atenci\u00f3n Primaria y trabajo social del \u00e1rea para determinar la seguridad en el domicilio.", "2": "Evaluaci\u00f3n formal de la capacidad para tomar decisiones concernientes a su salud.", "3": "Evaluaci\u00f3n para descartar demencia.", "4": "Evaluaci\u00f3n para descartar depresi\u00f3n.", "5": NaN}, "correct_option": 2, "explanations": {"1": {"exist": false, "char_ranges": [], "word_ranges": [], "text": ""}, "2": {"exist": true, "char_ranges": [[186, 279]], "word_ranges": [[31, 46]], "text": "es una medida de protecci\u00f3n de los pacientes m\u00e1s fr\u00e1giles para asegurar los mejores cuidados."}, "3": {"exist": false, "char_ranges": [], "word_ranges": [], "text": ""}, "4": {"exist": false, "char_ranges": [], "word_ranges": [], "text": ""}, "5": {"exist": false, "char_ranges": [], "word_ranges": [], "text": ""}}} {"id": 577, "year": 2022, "question_id_specific": 105, "full_question": "Var\u00f3n de 45 a\u00f1os tra\u00eddo a la consulta por sus familiares por un cuadro de un a\u00f1o de evoluci\u00f3n de p\u00e9rdida de memoria, que ha ido progresando hasta necesitar ayuda para algunas actividades de su vida cotidiana. Tiene antecedentes de migra\u00f1as y alg\u00fan episodio de focalidad neurol\u00f3gica autolimitada por los que no ha consultado nunca. La exploraci\u00f3n neuropsicol\u00f3gica es compatible con una demencia inicial y la RM cerebral muestra una leucoencefalopat\u00eda grave. \u00bfQu\u00e9 prueba habr\u00eda que solicitar?:", "full_answer": "Describen un caso caracter\u00edstico de CADASIL (Cerebral Autosomal Dominant Arteriopathy with Sub-cortical Infarcts and Leukoencephalopathy). Es una enfermedad gen\u00e9tica, de herencia autos\u00f3mica dominante por mutaci\u00f3n del gen NOTCH3. Esta mutaci\u00f3n provoca afectaci\u00f3n vascular de peque\u00f1o vaso que genera leucoencefalopat\u00eda grave de forma precoz, lo que genera sintomatolog\u00eda variada. Destacan las migra\u00f1as, la enfermedad cerebrovascular y deterioro cognitivo a edades tempranas, tal y como muestra el caso cl\u00ednico de la pregunta.", "type": "NEUROLOG\u00cdA", "options": {"1": "Punci\u00f3n lumbar para analizar amiloide y prote\u00edna tau en LCR.", "2": "Estudio gen\u00e9tico para presenilina 1 (PSEN1).", "3": "Estudio gen\u00e9tico para NOTCH3.", "4": "Estudio gen\u00e9tico para progranulina.", "5": NaN}, "correct_option": 3, "explanations": {"1": {"exist": false, "char_ranges": [], "word_ranges": [], "text": ""}, "2": {"exist": false, "char_ranges": [], "word_ranges": [], "text": ""}, "3": {"exist": true, "char_ranges": [[0, 228]], "word_ranges": [[0, 28]], "text": "Describen un caso caracter\u00edstico de CADASIL (Cerebral Autosomal Dominant Arteriopathy with Sub-cortical Infarcts and Leukoencephalopathy). Es una enfermedad gen\u00e9tica, de herencia autos\u00f3mica dominante por mutaci\u00f3n del gen NOTCH3."}, "4": {"exist": false, "char_ranges": [], "word_ranges": [], "text": ""}, "5": {"exist": false, "char_ranges": [], "word_ranges": [], "text": ""}}} {"id": 615, "year": 2022, "question_id_specific": 118, "full_question": "Var\u00f3n de 41 a\u00f1os que consulta por gonalgia de varios d\u00edas de evoluci\u00f3n. En la exploraci\u00f3n se realiza el test de Thessaly (dolor con los movimientos de rotaci\u00f3n interna y externa con la rodilla flexionada) que resulta positivo. \u00bfCu\u00e1l de las siguientes lesiones es m\u00e1s probable?:", "full_answer": "Respuesta 1 correcta: El test de Thessaly es una maniobra de exploraci\u00f3n meniscal. El enunciado se centra exclusivamente en esta parte de la exploraci\u00f3n y nos explica, adem\u00e1s, que efectivamente es positiva lo que hace sospechar que existe lesi\u00f3n meniscal. \"The Thessaly test is a dynamic reproduction of load transmission in the knee joint and is performed at 5\u00b0 and 20\u00b0 of flexion. The examiner supports the patient by holding his or her outstretched hands while the patient stands flatfooted on the floor. The patient then rotates his or her knee and body, internally and externally, three times, keeping the knee in slight flexion (5\u00b0). Then the same procedure is carried out with the knee flexed at 20\u00b0. Patients with suspected meniscal tears experience medial or lateral joint-line discomfort and may have a sense of locking or catching. The theory behind the test is that, with this maneuver, the knee with a meniscal tear is subjected to excessive loading conditions and almost certainly will have the same symptoms that the patient reported. The test is always performed first on the normal knee so that the patient may be trained, especially with regard to how to keep the knee in 5\u00b0 and then in 20\u00b0 of flexion and how to recognize, by comparison, a possible positive result in the symptomatic knee.\" Respuesta 2 incorrecta: No nos describen en ning\u00fan momento maniobras de exploraci\u00f3n del ligamento cruzado anterior, tampoco nos hablan de un traumatismo que lo haya podido lesionar. Respuesta 3 incorrecta: No nos describen en ning\u00fan momento maniobras de exploraci\u00f3n del ligamento cruzado posterior, tampoco nos hablan de un traumatismo que lo haya podido lesionar. Respuesta 4 incorrecta: Aunque podr\u00eda darse no es lo esperable en una persona de 41 a\u00f1os con una cl\u00ednica que se ha establecido de forma aguda.", "type": "TRAUMATOLOG\u00cdA", "options": {"1": "Lesi\u00f3n meniscal.", "2": "Lesi\u00f3n por rotura del ligamento cruzado anterior.", "3": "Lesi\u00f3n por rotura del ligamento cruzado posterior.", "4": "Lesi\u00f3n por artropat\u00eda degenerativa.", "5": NaN}, "correct_option": 1, "explanations": {"1": {"exist": true, "char_ranges": [[0, 255]], "word_ranges": [[0, 40]], "text": "Respuesta 1 correcta: El test de Thessaly es una maniobra de exploraci\u00f3n meniscal. El enunciado se centra exclusivamente en esta parte de la exploraci\u00f3n y nos explica, adem\u00e1s, que efectivamente es positiva lo que hace sospechar que existe lesi\u00f3n meniscal."}, "2": {"exist": true, "char_ranges": [[1310, 1491]], "word_ranges": [[220, 247]], "text": "Respuesta 2 incorrecta: No nos describen en ning\u00fan momento maniobras de exploraci\u00f3n del ligamento cruzado anterior, tampoco nos hablan de un traumatismo que lo haya podido lesionar."}, "3": {"exist": true, "char_ranges": [[1492, 1674]], "word_ranges": [[247, 274]], "text": "Respuesta 3 incorrecta: No nos describen en ning\u00fan momento maniobras de exploraci\u00f3n del ligamento cruzado posterior, tampoco nos hablan de un traumatismo que lo haya podido lesionar."}, "4": {"exist": true, "char_ranges": [[1675, 1817]], "word_ranges": [[274, 300]], "text": "Respuesta 4 incorrecta: Aunque podr\u00eda darse no es lo esperable en una persona de 41 a\u00f1os con una cl\u00ednica que se ha establecido de forma aguda."}, "5": {"exist": false, "char_ranges": [], "word_ranges": [], "text": ""}}} {"id": 456, "year": 2018, "question_id_specific": 200, "full_question": "Hombre de 67 a\u00f1os de edad, diab\u00e9tico y asm\u00e1tico, con un glaucoma primario de \u00e1ngulo abierto en ambos ojos. Presenta una presi\u00f3n ocular de 34 mmHg en su ojo derecho y 31 mmHg en su ojo izquierdo. \u00bfCu\u00e1l de los siguientes grupos farmacol\u00f3gicos es el m\u00e1s indicado como primera elecci\u00f3n como tratamiento hipotensor?", "full_answer": "Nos preguntan por el tratamiento de primera elecci\u00f3n de un glaucoma. Antiguamente la primera elecci\u00f3n eran los betabloqueantes, pero actualmente han sido desplazados por el grupo que m\u00e1s baja la presi\u00f3n intaocular: las prostaglandinas. Su eficacia y la posolog\u00eda m\u00e1s c\u00f3moda (una vez al d\u00eda), los hacen en general el grupo de elecci\u00f3n. Los betabloqueantes quedar\u00edan como segunda elecci\u00f3n en casos en los que los efectos secundarios de las prostaglandinos (cambio de color de ojos, aumento del tama\u00f1o de las pesta\u00f1as) nos hagan dejarlas inicialmente de lado. Debido a que el paciente es asm\u00e1tico, deber\u00edamos descartar los betabloqueantes (respuesta 2 incorrecta). Adem\u00e1s como necesitamos una bajada importante de presi\u00f3n intraocular porque las cifras son altas (34 y 31), podemos tambi\u00e9n descartar los inhibidores de la anhidrasa carb\u00f3nica y los agonistas alfa-2 (opciones 1 y 3 incorrectas). Y decantarnos por el grupo que m\u00e1s puede bajar la presi\u00f3n intraocular. Respuesta 4 correcta. Y luego un par de matices del enunciado. Ser\u00eda m\u00e1s correcto hablar de presi\u00f3n intraocular y no de \u201cpresi\u00f3n ocular\u201d. Y la redacci\u00f3n podr\u00eda mejorarse para no repetir la palabra \u201ccomo\u201d en la \u00faltima frase.", "type": "OFTALMOLOG\u00cdA", "options": {"1": "Inhibidores de la anhidrasa carb\u00f3nica.", "2": "Beta-bloqueantes.", "3": "Agonistas alfa-2.", "4": "Derivados de prostaglandinas.", "5": NaN}, "correct_option": 4, "explanations": {"1": {"exist": true, "char_ranges": [[662, 890]], "word_ranges": [[101, 136]], "text": "Adem\u00e1s como necesitamos una bajada importante de presi\u00f3n intraocular porque las cifras son altas (34 y 31), podemos tambi\u00e9n descartar los inhibidores de la anhidrasa carb\u00f3nica y los agonistas alfa-2 (opciones 1 y 3 incorrectas)."}, "2": {"exist": true, "char_ranges": [[557, 661]], "word_ranges": [[87, 101]], "text": "Debido a que el paciente es asm\u00e1tico, deber\u00edamos descartar los betabloqueantes (respuesta 2 incorrecta)."}, "3": {"exist": true, "char_ranges": [[662, 890]], "word_ranges": [[101, 136]], "text": "Adem\u00e1s como necesitamos una bajada importante de presi\u00f3n intraocular porque las cifras son altas (34 y 31), podemos tambi\u00e9n descartar los inhibidores de la anhidrasa carb\u00f3nica y los agonistas alfa-2 (opciones 1 y 3 incorrectas)."}, "4": {"exist": true, "char_ranges": [[69, 235]], "word_ranges": [[11, 34]], "text": "Antiguamente la primera elecci\u00f3n eran los betabloqueantes, pero actualmente han sido desplazados por el grupo que m\u00e1s baja la presi\u00f3n intaocular: las prostaglandinas."}, "5": {"exist": false, "char_ranges": [], "word_ranges": [], "text": ""}}} {"id": 484, "year": 2020, "question_id_specific": 175, "full_question": "Un hombre de 18 a\u00f1os llega al servicio de urgencias del hospital trasladado por una unidad de soporte vital b\u00e1sico. Ha sido atropellado por un autom\u00f3vil cuando iba en bicicleta. A su llegada tiene una frecuencia card\u00edaca de 115 lpm, una presi\u00f3n arterial de 110/75 mmHg, pulso radial palpable, un relleno capilar de 2,5 segundos y una frecuencia respiratoria de 25 rpm. \u00bfQu\u00e9 nos indican estos datos?:", "full_answer": "T\u00e9cnicamente el paciente se encuentra en un grado II de hemorragia seg\u00fan la clasificaci\u00f3n de la ATLS, en principio esto nos habla de una p\u00e9rdida de entre 750 y 1500mL de sangre y la actuaci\u00f3n indicada es reanimaci\u00f3n con cristaloides; no obstante, en las gu\u00edas de atenci\u00f3n al paciente politraumatizado grave, cada vez se preconiza m\u00e1s el uso precoz de hemoderivados, sobre todo en casos l\u00edmite como este, y en pacientes j\u00f3venes, que pueden no llegar a traducir la gravedad de la p\u00e9rdida sangu\u00ednea hasta que esta se hace mucho m\u00e1s significativa. Por tanto dudar\u00eda entre las opciones 1 y 2.", "type": "CUIDADOS CR\u00cdTICOS", "options": {"1": "Ha perdido entre un 15 y un 30 % de la volemia y precisa reposici\u00f3n de volumen.", "2": "Ha perdido m\u00e1s de un 30 % de la volemia y precisa reposici\u00f3n de volumen y de sangre.", "3": "Ha perdido m\u00e1s de un 40 % de la volemia y precisar\u00e1 tratamiento quir\u00fargico.", "4": "Ha perdido m\u00e1s de un 50% de la volemia y precisa reposici\u00f3n de volumen y de sangre.", "5": NaN}, "correct_option": 1, "explanations": {"1": {"exist": false, "char_ranges": [], "word_ranges": [], "text": ""}, "2": {"exist": false, "char_ranges": [], "word_ranges": [], "text": ""}, "3": {"exist": false, "char_ranges": [], "word_ranges": [], "text": ""}, "4": {"exist": false, "char_ranges": [], "word_ranges": [], "text": ""}, "5": {"exist": false, "char_ranges": [], "word_ranges": [], "text": ""}}}